Sunteți pe pagina 1din 64

ISSN 2601-0305

ISSN-L 2601-0305

REVISTA DE MATEMATICĂ
MARINESCU-GHEMECI OCTAVIAN
PUBLICAT, IE ANUALĂ
PENTRU ELEVI S, I PROFESORI

Liceul ,,S, tefan Diaconescu” Potcoava

Anul IV, nr. 1/2020

Editura Hoffman
Editori:
Costel BĂLCĂU (redactor s, ef) Costel ANGHEL (redactor)
Florea BADEA (redactor) Mihai Florea DUMITRESCU
(secretar general de redact, ie)
Comitetul s, tiint, ific:
Membri de onoare:
Prof. univ. dr. Victor ALEXANDRU - Universitatea din Bucures, ti
Prof. univ. dr. Tudor BĂLĂNESCU - Universitatea din Pites, ti
Prof. univ. dr. Corneliu UDREA - Universitatea din Pites, ti
Membri:
Stelian-Corneliu ANDRONESCU - Universitatea din Pites, ti
Costel BĂLCĂU - Universitatea din Pites, ti
Doru CONSTANTIN - Universitatea din Pites, ti
Cezar JOIT, A - I.M.A.R.
Ruxandra MARINESCU-GHEMECI - Universitatea din Bucures, ti
Radu MICULESCU - Universitatea Transilvania din Bras, ov
Cristinel MORTICI - Universitatea Valahia din Târgovis, te
Cristian NICULESCU - Universitatea din Bucures, ti
Comitetul de redact, ie:
Leonard GIUGIUC Marian HAIDUCU Marin IONESCU
Daniel JINGA Marius MÂINEA Marius PERIANU
Florin STĂNESCU Adrian T, URCANU Sorin ULMEANU

Colaboratori:
Delia Ileana BASCH-NAIDIN Eduard BUZDUGAN Luigi Ionut, CATANA
Aurel CHIRIT , Ă Dumitru ILIE Lavinia DUMITRANA
Iuliana ION-IONESCU Oana KRISZTA Cosmin MANEA
Ileana MARINESCU-GHEMECI Iulia MARINESCU-GHEMECI George MIHAI
Adriana MIU Corina Mianda MÎINESCU Constantin MOGOS , ANU
Florin NĂSUI Mariana NĂSUI Grat, iela POPA
Dorin POPA Sorin PELIGRAD Dragos, PETRICĂ
Florina PETRUS , CĂ Valentin RĂDULESCU Cristina SMARANDACHE
Valentin SMARANDACHE Manuela STROE Daniela TACLIT
Marian TELER Nicolae TOMESCU Iuliana TRAS , CĂ

Tehnoredactare computerizată: Mihail TĂNASE, e-mail: mihaimit@yahoo.it


Redact, ia: Liceul ,,S, tefan Diaconescu”, Str. Principală, nr. 197, cod 237355,
Potcoava, tel. 0742123058, e-mail: florin14mihai@yahoo.com
Revista este editată ı̂n colaborare cu Departamentul de Matematică-Informatică,
Universitatea din Pites, ti.
Forma digitală a revistei poate fi accesată la adresa: http://rmgo.upit.ro
Publicată de: Editura Hoffman, www.EdituraHoffman.com, www.LibrariaHoffman.ro
Anul IV, Nr. 1, 2020
PROBLEME PENTRU CONCURSURI 3

PROBLEME PENTRU CONCURSURI

Probleme propuse

Clasa a V-a

MGO 121. Determinat, i numerele naturale abc cu a, b s, i c cifre distincte, astfel


ı̂ncât abc = (2a + 2b + 2c)2 .

Aurel Dobos, an, Lugoj

MGO 122. Fie s, irul de numere 1, 3, 32 , 33 , . . ..

a) Arătat, i că numărul 22019 nu poate fi scris ca sumă de elemente diferite din
acest s, ir.

b) Dar numărul 22020 ?

Nicolae Stăniloiu, Bocs, a

MGO 123. Arătat, i că oricum s-ar alege nouă numere naturale impare mai mici
decât 50, există trei dintre ele care să aibă acelas, i număr de divizori.

***

MGO 124. a) Există două numere naturale prime a căror diferent, ă să fie egală
cu 6 s, i a căror sumă să fie un număr de forma 24k+3 , k ∈ N?

b) Dar cu suma de forma 24k+2 , k ∈ N?

Mihai Florea Dumitrescu, Potcoava

MGO 125. a) Arătat, i că numerele 36 + 63 s, i 36 · 63 + 1 sunt divizibile cu 7.

b) Fie n ∈ N. Demonstrat, i că 3n + n3 este divizibil cu 7 dacă s, i numai dacă


3n · n3 + 1 este divizibil cu 7.

Costel Anghel, Slatina s, i Florea Badea, Scornices, ti


4 Probleme propuse

Clasa a VI-a

MGO 126. Determinat, i numerele naturale n pentru care numărul 2020n − 2019n
este divizibil cu 7.

Ionel Tudor, Călugăreni

MGO 127. Rezolvat, i ı̂n numere ı̂ntregi ecuat, ia x15 + y 9 = 2020.

Stelian Corneliu Andronescu s, i Costel Bălcău, Pites, ti

MGO 128. Fie a, b, c ∈ Q s, i n ∈ N∗ astfel ı̂ncât a + b =


6 0, b + c 6= 0 s, i
a+b c−a n2 + n + 1
(n + 1)a + b = nc. Arătat, i că + = .
b+c a+b n(n + 1)

Marin Chirciu, Pites, ti

MGO 129. Fie triunghiul isoscel ABC cu AB = AC s, i ^A = 20◦ . Se construiesc


semidreptele s1 , s2 , . . . , sm , m ∈ N∗ , fiecare având originea ı̂n punctul B s, i fiind
interioară unghiului ^ABC, astfel ı̂ncât unghiul dintre semidreapta si s, i semi-
dreapta [BC să aibă măsura egală cu i · 5◦ , pentru orice i ∈ {1, 2, . . . , m}. Analog,
se construiesc semidreptele t1 , t2 , . . . , tn , n ∈ N∗ , fiecare având originea ı̂n punctul
C s, i fiind interioară unghiului ^ACB, astfel ı̂ncât unghiul dintre semidreapta tj
s, i semidreapta [CB să aibă măsura egală cu j · 3◦ , pentru orice j ∈ {1, 2, . . . , n}.
Oricare două semidrepte si s, i tj , cu i ∈ {1, 2, . . . , m} s, i j ∈ {1, 2, . . . , n}, se inter-
sectează ı̂ntr-un punct; acest punct ı̂mpreună cu punctele B s, i C determină un
triunghi. Notăm cu T mult, imea tuturor acestor triunghiuri.
a) Determinat, i cele mai mari valori posibile pentru numerele m s, i n.
b) Pentru m s, i n determinate la punctul a), calculat, i probabilitatea ca alegând
la ı̂ntâmplare un triunghi din mult, imea T , acesta să fie isoscel sau dreptunghic.

Costel Anghel, Slatina s, i Florea Badea, Scornices, ti

MGO 130. Se consideră un triunghi ABC cu AB = AC, E mijlocul laturii BC s, i


D un punct oarecare pe segmentul (AE). Fie DM ⊥ AB, M ∈ AB s, i M N ⊥ AC,
N ∈ AE. Demonstrat, i că punctul N este mijlocul segmentului (AD) dacă s, i numai
dacă 4ABC este echilateral.

Mihai Florea Dumitrescu, Potcoava


Probleme propuse 5

Clasa a VII-a

MGO 131. Determinat, i perechile (x, y) de numere naturale nenule cu proprietatea


că
x2 + y 2 · [7x, y] − 91xy = 0,


unde [7x, y] reprezintă cel mai mic multiplu comun al numerelor 7x s, i y.

Stelian Corneliu Andronescu s, i Costel Bălcău, Pites, ti

MGO 132. Demonstrat, i că fract, ia

12019 + 22019 + 32019 + 42019 + 52019 + 62019


12020 + 22020 + 32020 + 42020 + 52020 + 62020
este reductibilă.

Ionel Tudor, Călugăreni

MGO 133. Determinat, i numerele prime p s, i q, p > 3, astfel ı̂ncât

(p − 3)! + 9
− 4225 = q 8 ,
p−2

unde n! = 1 · 2 · 3 · . . . · n, pentru orice n ∈ N∗ .

Costel Anghel, Slatina s, i Florea Badea, Scornices, ti

MGO 134. Se consideră un triunghi neisoscel ABC. Fie G centrul său de greutate
s, i I centrul cercului ı̂nscris ı̂n acest triunghi. Arătat, i că dacă GI ⊥ BC, atunci
b+c
a= , unde a, b s, i c reprezintă lungimile laturilor BC, AC s, i respectiv AB.
3
Reciproca este adevărată?

Nicolae Stăniloiu, Bocs, a

MGO 135. Fie ABCD un patrulater cu AB = BC = CD, m (^ABC) = 70◦ s, i


m (^BCD) = 170◦ .
a) Calculat, i măsura unghiului dintre diagonalele patrulaterului.
b) Calculat, i măsurile unghiurilor BAD s, i ADC ale patrulaterului.

Costel Anghel, Slatina s, i Florea Badea, Scornices, ti


6 Probleme propuse

Clasa a VIII-a

MGO 136. Pentru orice număr natural nenul n considerăm mult, imea
(r r r r )
1 2 3 n
An = , , ,..., .
2020 2020 2020 2020

a) Câte numere n ∈ N∗ au proprietatea că An cont, ine exact 2020 de numere


rat, ionale?
b) Dar exact 2020 de numere irat, ionale?
Stelian Corneliu Andronescu s, i Costel Bălcău, Pites, ti
1
MGO 137. Pentru orice n ∈ N∗ considerăm numerele an = s, i
n(n + 1)
1 1 1 1
Sn = + + + ... + .
a1 a1 + a2 a1 + a2 + a3 a1 + a2 + a3 + . . . + an
Arătat, i că 68 < S64 < 71.
Costel Anghel, Slatina s, i Florea Badea, Scornices, ti

MGO 138. Arătat, i că pentru orice x1 ≥ 1, x2 ≥ 2 s, i x3 ≥ 3 astfel ı̂ncât


1 2 3
+ + = s, s ∈ (0, 3), are loc inegalitatea
x1 x2 x3
p √ √ √
(3 − s)(x1 + x2 + x3 ) ≥ x1 − 1 + x2 − 2 + x3 − 3.
Dorin Mărghidanu, Corabia

MGO 139. Se consideră piramida hexagonală regulată V ABCDEF cu muchia


bazei AB = a. Calculat, i ı̂nălt, imea acestei piramide (ı̂n funct, ie de a), s, tiind că aria
sa laterală este egală cu aria totală a piramidei V ACE.
Costel Anghel, Slatina

MGO 140. Se consideră piramida hexagonală regulată V ABCDEF cu muchia


bazei AB = 9 cm s, i ı̂nălt, imea V O = 12 cm. Dintre toate prismele hexagonale
regulate ı̂nscrise ı̂n piramida dată, se consideră aceea de volum maxim. Calculat, i:
a) raportul dintre volumul acestei prisme s, i volumul piramidei;
b) ı̂nălt, imea prismei;
c) distant, a de la V la vârfurile prismei situate pe fet, ele laterale ale piramidei.
Florea Badea, Scornices, ti
Probleme propuse 7

Clasa a IX-a

MGO 141. Rezolvat, i ı̂n mult, imea numerelor reale următorul sistem de ecuat, ii:
(
a2 + b2 + c2 = 1
√ √ .
|a + 4b − 5c| + |b + 4c − 5a| + |c + 4a − 5b| + 14(a + b + c)2 = 3 14

Do Xuan Trong, Vietnam

MGO 142. Fie a ∈ N∗ . Rezolvat, i ı̂n mult, imea numerelor reale ecuat, ia
   
x x x
− = .
a a+1 2a + 1

Marin Chirciu, Pites, ti

MGO 143. Fie a, b, c, d numere reale pozitive astfel ı̂ncât ab+bc+cd+da = 4abcd.
Demonstrat, i că

1 1 1 1 √
√ +√ +√ +√ ≤ 2 2.
a3 + b3 b3 + c3 c3 + d3 d3 + a3

Daniel Jinga, Pites, ti

MGO 144. Demonstrat, i că ı̂n orice triunghi ABC are loc identitatea
 
A B C A B C
(p − a) sin + (p − b) sin + (p − c) sin = r cos + cos + cos .
2 2 2 2 2 2

Mihály Bencze, Bras, ov

MGO 145. În triunghiul ABC se ı̂nscrie un semicerc care are centrul pe latura
BC s, i este tangent la laturile AB s, i AC. Se consideră punctele Y ∈ [AC] s, i
Z ∈ [AB] astfel ı̂ncât dreapta Y Z să fie tangentă la semicerc. Fie CY = y s, i
BZ = z. Demonstrat, i că

(b + c)2 yz = b2 − c2 (bz − cy) + a2 bc.




Francisco Javier Garcı́a Capitán, Spania


8 Probleme propuse

Clasa a X-a

MGO 146. Demonstrat, i că pentru orice n ∈ N∗ are loc inegalitatea


n
X 1 n
n+1 < .
(k 2 + k) +n (n + 1)2
k=1
Mihály Bencze, Bras, ov
MGO 147. Rezolvat, i ı̂n mult, imea numerelor reale ecuat, ia
3 √
q p3
13x − 2x − 3x − 3 · 6x−1 − 21x + 2x + 3x + 3 · 6x−1 − 21x − 1
3 √
q p3
= 13x + 3 · 6x−1 − 21x − 1 − 3 · 6x−1 + 21x.
Sorin Ulmeanu, Pites, ti
MGO 148. Fie z1 , z2 s, i z3 numere complexe ce satisfac relat, iile
1 1 1
1 + z1 z2 z3 = 0 s, i z1 + z2 + z3 = + + .
z1 z2 z3
Demonstrat, i că produsul a două dintre numerele z1 , z2 , z3 este egal cu 1.
Dorin Mărghidanu, Corabia
MGO 149. Fie s, irurile (an )n≥1 s, i (bn )n≥1 definite prin
π 3π (2n − 1)π
an = sin + sin + . . . + sin ,
2n + 1 2n + 1 2n + 1
π 3π (2n − 1)π
bn = cos + cos + . . . + cos ,
2n + 1 2n + 1 2n + 1
pentru orice n ∈ N∗ .
a) Studiat, i monotonia celor două s, iruri.
b) Arătat, i că an > bn , pentru orice n ∈ N∗ .
Mihai Florea Dumitrescu, Potcoava
MGO 150. Fie d1 s, i d2 două drepte perpendiculare care se intersectează ı̂n
punctul O s, i fie ω1 , ω2 , ω3 s, i ω4 patru cercuri care trec prin O, astfel ı̂ncât ω1 s, i
ω2 sunt tangente la d1 iar ω3 s, i ω4 sunt tangente la d2 .
Demonstrat, i că, indiferent cum s-ar nota celelalte patru puncte de intersect, ie
ale cercurilor date cu X, Y , Z s, i T , avem
OX OZ XZ
· = .
OY OT YT
Cristinel Mortici, Viforâta
Probleme propuse 9

Clasa a XI-a

MGO 151. Fie A, B, C ∈ M2 (R) astfel ı̂ncât AB + BC + CA = BA + CB + AC.


Demonstrat, i că

 3
det A2 + B 2 + C 2 − AB − BC − CA ≥ [det(A − C) − det(B − C)]2 .
4

Daniel Jinga, Pites, ti

MGO 152. Fie A, B ∈ Mn (C), unde n ∈ N, n ≥ 2, astfel ı̂ncât A2 = On ,


rang (A) = 1 s, i B 2 = AB 6= On .

a) Demonstrat, i că n > 2 s, i BA = On .

b) Arătat, i că pentru orice n > 2 există o infinitate de matrice cu proprietăt, ile
date.

Florin Stănescu, Găes, ti

MGO 153. Fie s, irul (xn )n≥0 definit prin x0 = 3 s, i xn = 2xn−1 + n sin n, ∀ n ≥ 1.

a) Arătat, i că xn > 0, pentru orice n ∈ N.



b) Demonstrat, i că s, irul (yn )n≥2 definit prin yn = n xn este convergent s, i
calculat, i-i limita.

Stelian Corneliu Andronescu s, i Costel Bălcău, Pites, ti


 
1
MGO 154. Fie x, y ∈ 0, astfel ı̂ncât 2x + y 3 < 2y + x3 .
2
Demonstrat, i că 3x + y 2 < 3y + x2 s, i reciproc.

Cristinel Mortici, Viforâta

MGO 155. Fie x, y, z ≥ 0 astfel ı̂ncât xy 2 + yz 2 + zx2 + xyz ≤ 4.

Demonstrat, i că x + y + z ≥ xy + yz + zx.

Leonard Mihai Giugiuc, România s, i Hung Nguyen Viet, Vietnam


10 Probleme propuse

Clasa a XII-a

MGO 156. Fie p un număr prim de forma p = 4k + 3, k ∈ N∗ . Demonstrat, i că


 2
ecuat, ia x2 + b
k +b
4 =b9 nu are solut, ii ı̂n Zp .

Stelian Corneliu Andronescu s, i Costel Bălcău, Pites, ti

MGO 157. Fie n ∈ N, n ≥ 3. Rezolvat, i ı̂n Rn+ sistemul


 5

 x1 + 3 = x22 + 3x3

 5
x2 + 3 = x23 + 3x4 .


 ...
 5
xn + 3 = x21 + 3x2
Mihály Bencze, Bras, ov

MGO 158. Fie n ∈ N∗ , a ∈ R∗ s, i b > 0. Calculat, i integrala

x2n−1 (x + a)2n−1
Z
dx, x ∈ (0, ∞).
(x + a)4n + bx4n
Daniel Jinga, Pites, ti

MGO 159. Fie f : [−1, 1] → R o funct, ie de două ori derivabilă, cu derivata a


doua continuă, astfel ı̂ncât f (−1) = f (1) = 0. Demonstrat, i că are loc inegalitatea
Z 1
1 2
· f 00 (x) dx ≥ max f 2 (x).
6 −1 x∈[−1,1]

Florin Stănescu, Găes, ti

MGO 160. Fie a, b, c, d ≥ 0 astfel ı̂ncât ab + ac + ad + bc + bd + cd = 6.


Demonstrat, i că

2 2 22+ 22
 √ 
a +b +c +d + · (abc + abd + acd + bcd) ≥ 2 4 + 2 .
2
Când are loc egalitatea?
Leonard Mihai Giugiuc, Drobeta Turnu Severin
Rezolvarea problemelor din numărul anterior 11

Rezolvarea problemelor din numărul anterior

Clasa a V-a

MGO 81. În câte moduri se poate scrie 2020 ca sumă de numere impare conse-
cutive?

Costel Anghel, Slatina s, i Florea Badea, Scornices, ti

Solut, ie. Fie 2k + 1, 2k + 3, . . . , 2k + 2n − 1 n numere impare consecutive. Suma


lor este 2k + 1 + 2k + 3 + . . . + 2k + 2n − 1 = 2nk + n2 = n(n + 2k). Avem
n(n + 2k) = 2020, adică n(n + 2k) = 22 · 5 · 101. Cum n s, i n + 2k au aceeas, i paritate,
rezultă că n este un divizor par al lui 2020 s, i n nu se divide cu 4. Deci n poate fi 2,
10, 202 sau 1010. Pentru n = 2 găsim 2k = 1008, deci solut, ia 2020 = 1009 + 1011.
Pentru n = 10 găsim 2k = 192, deci solut, ia 2020 = 193 + 195 + . . . + 211. Pentru
n = 202 găsim 202 + 2k = 10, fals. Pentru n = 1010 găsim 1010 + 2k = 2, de
asemenea fals. Deci problema are două solut, ii.

MGO 82. Arătat, i că numerele 45n s, i 45n + 5n au acelas, i număr de cifre, pentru
orice număr natural n.

Costel Anghel, Slatina s, i Florea Badea, Scornices, ti

Solut, ie. Presupunând că numerele date nu au acelas, i număr de cifre, rezultă că
există p natural astfel ı̂ncât 45n < 10p ≤ 45n + 5n . Deducem că p > n s, i, ı̂mpărt, ind
prin 5n , că 9n < 2p · 5p−n ≤ 9n + 1. Dar numerele 9n s, i 9n + 1 sunt consecutive,
deci 2p · 5p−n = 9n + 1 = M8 + 2, de unde p = 1 s, i n = 0, care nu verifică egalitatea
2p · 5p−n = 9n + 1.

MGO 83. Determinat, i ultimele două cifre ale numărului N = 7A , unde


A = p21 + p22 + . . . + p22019 , p1 , p2 , . . . , p2019 fiind numere naturale impare.

Stelian Corneliu Andronescu s, i Costel Bălcău, Pites, ti

Solut, ie. Se s, tie că dacă p este impar avem p2 = M4 + 1, deci A = M4 + 3. Dar
zu(74k ) = 01, zu(74k+1 ) = 07, zu(74k+2 ) = 49, zu(74k+3 ) = 43, deci zu(A) = 43.

MGO 84. Se consideră numerele 1000, 1001, 1002, . . . , 2019.


a) Câte pătrate perfecte se găsesc printre numerele date?
b) Calculat, i suma resturilor ı̂mpărt, irilor numerelor date prin 13.

***
12 Rezolvarea problemelor din numărul anterior

Solut, ie. a) Cel mai mic pătrat perfect din s, irul dat este 1024 = 322 , iar cel mai
mare 1936 = 442 , deci avem 44 − 31 = 13 pătrate perfecte.
b) Avem 1000 = 76·13+12, 1001 = 77·13, 1002 = 77·13+1, . . ., 2015 = 155·13,
2016 = 155 · 13 + 1, . . ., 2019 = 155 · 13 + 4. De la 1001 la 2014 sunt 1014 : 13 = 78
grupe de câte 13 resturi, cu suma fiecărei grupe 0 + 1 + 2 + . . . + 12 = 78. Astfel
suma tuturor resturilor este 12 + 78 · 78 + 0 + 1 + 2 + 3 + 4 = 6106.

MGO 85. Câte numere de 3 cifre au proprietatea că prin adunare cu răsturnatul
se obt, ine un număr de 4 cifre identice?
Mihai Florea Dumitrescu, Potcoava
Solut, ie. Avem abc + cba = xxxx, adică 101a + 20b + 101c = x · 11 · 101. Rezultă
că 20b = M101, de unde b = 0. Atunci a + c = 11x, deci x = 1 s, i a + c = 11.
Obt, inem numerele 209, 308, 407, 506, 605, 704, 803 s, i 902, deci sunt 8 numere cu
proprietatea din enunt, .

Clasa a VI-a

MGO 86. Fie A o mult, ime de numere naturale care satisface condit, iile:
i) 1 ∈ A;
ii) Dacă x ∈ A, atunci 5x ∈ A;
iii) Dacă 7x − 1 ∈ A, atunci x ∈ A.
Arătat, i că 13 ∈ A.
Marin Chirciu, Pites, ti
Solut, ie. Din i) s, i ii) rezultă că 5, 25, 125 ∈ A. Cum 125 = 7 · 18 − 1 ∈ A, din iii)
rezultă că 18 ∈ A, deci conform ii) s, i 90 = 5 · 18 ∈ A. Cum 90 = 7 · 13 − 1 ∈ A,
din iii) rezultă că 13 ∈ A.

MGO 87. a) Determinat, i ultimele cinci cifre ale numărului 72019 , atunci când
este scris ı̂n sistemul binar.
b) Aceeas, i cerint, ă pentru numărul N = 12019 + 22019 + . . . + 20192019 .
Stelian Corneliu Andronescu s, i Costel Bălcău, Pites, ti
Solut, ie. a) Avem, succesiv: (2k + 1)2 = M8 + 1, (2k + 1)4 = M16 + 1, (2k + 1)8 =
M32 + 1. Rezultă că (2k + 1)2019 = (2k + 1)8·252+3 = (M32 + 1)(2k + 1)3 .
Astfel 72019 = (M32 + 1) · 73 = M32 + 23. Cum 23 = 10111(2) , rezultă că
72019 = . . . 10111(2) .
Rezolvarea problemelor din numărul anterior 13

b) Deoarece (M32 + i)2019 + (M32 − i)2019 = M32, rezultă că 12019 + 22019 +
. . . + 312019 + 322019 = M32, 332019 + 342019 + . . . + 632019 + 642019 = M32, . . . ,
19852019 + 19862019 + . . . + 20152019 + 20162019 = M32, deci A = M32 + 20172019 +
20182019 + 20192019 = M32 + 12019 + 22019 + 32019 = M32 + 1 + 33 = M32 + 28,
prin urmare A = . . . 11100(2) .

MGO 88. Se consideră numerele a, b, c, d ∈ Q∗ astfel ı̂ncât


a 3b 4c 5d
= = = 6= −1.
3b + 4c + 5d a + 4c + 5d a + 3b + 5d a + 3b + 4c
 
1 1 1 1
Calculat, i (a + b + c + d) + + + .
a b c d
Costel Anghel, Slatina s, i Florea Badea, Scornices, ti
Solut, ie. Adunând
 numărătoriila numitori,
 se deducecă a = 3b = 4c = 5d. Astfel
1 1 1 1 1 1 1 1 1391
(a + b + c + d) + + + =a 1+ + + · · (1 + 3 + 4 + 5) = .
a b c d 3 4 5 a 60
MGO 89. Fie ABC un triunghi echilateral s, i punctele D ∈ (BC), P ∈ (AB)
astfel ı̂ncât m (^BAD) = m (^ADP ) = x. Determinat, i valoarea lui x astfel ı̂ncât
perpendiculara din punctul P pe dreapta AD să treacă prin mijlocul laturii AC.
Mihai Florea Dumitrescu, Potcoava
Solut, ie. Fie M mijlocul laturii AC. Triunghiul AP D este isoscel s, i P M ⊥ AD,
deci P M este mediatoarea lui [AD]. Avem M D = M A = M C, deci triunghiul
ADC este dreptunghic ı̂n D. Rezultă că x = 30◦ .

MGO 90. Fie triunghiul ABC cu AB = AC s, i m (^A) = 80◦ . Se consideră


punctul D ı̂n semiplanul determinat de dreapta AB s, i care nu cont, ine punctul C,
astfel ı̂ncât m (^BCD) = 40◦ s, i CD = BC. Calculat, i măsura unghiului BAD.
Costel Anghel, Slatina s, i Florea Badea, Scornices, ti
Solut, ie. Construim triunghiul echilateral EBC, cu punctul E de aceeas, i parte a
dreptei BC ca s, i punctul A. Atunci dreapta EA este mediatoarea lui [BC] s, i CE =
BC = CD. Rezultă că 4DCE este isoscel. Cum m (^ACD) = m (^ACE) = 10◦ ,
rezultă că CA este bisectoarea unghiului DCE, deci este s, i mediatoarea lui [DE]
Astfel AD = AE. Rezultă că m (^DAC) = m (^EAC) = 180◦ −(30◦ +10◦ ) = 140◦ ,
prin urmare m (^BAD) = m (^DAC) − m (^BAC) = 140◦ − 80◦ = 60◦ .
14 Rezolvarea problemelor din numărul anterior

Clasa a VII-a

MGO 91. Determinat, i perechile (x, y) de numere ı̂ntregi cu proprietatea că


1255 + x2 = 2y .
Costel Anghel, Slatina s, i Florea Badea, Scornices, ti
Solut, ie. Se observă că y > 10 s, i x este impar. Rezultă că u(x2 ) ∈ {1, 5, 9},
deci u(2y ) ∈ {4,6}, deci y este par. Fie y = 2k, k > 5. Ecuat, ia dată devine
2k − x 2k + x = 1255. Dar 1255 = 5 · 251, iar 251 este prim. Cum 2k − x +

 k
k
 k+1 2 −x=5
2 + x = 2 , singurele cazuri favorabile sunt , cu solut, ia
2k + x = 251
 k
2 − x = 251
k = 7, x = 123, s, i , cu solut, ia k = 7, x = −123. În concluzie,
2k + x = 5
(x, y) ∈ {(123, 14), (−123, 14)}.
2 2
MGO r 92. Fie numerele rat, ionale pozitive x s, i y astfel ı̂ncât 2(x − y) + 4y = 3xy
11x + 3y 2x + 3y
s, i ∈ Q. Calculat, i valoarea raportului .
7x + 2y 4x + 5y
Marin Chirciu, Pites, ti
Solut, ie. Efectuând calculele, egalitatea dată devine 2x2 − 7xy + 6y 2 = 0, sau,
echivalent, (x−2y)(2x−3y)
√ = 0, deci x = 2y sau 2x = 3y. Pentru 2x = 3y obt, inem
r r
11x + 3y 39 11x + 3y 5
că = , fals. Pentru x = 2y obt, inem că = ∈ Q.
7x + 2y 5 7x + 2y 4
2x + 3y 7
Rezultă că x = 2y, de unde = .
4x + 5y 13
MGO 93. Determinat, i n ∈ N∗ pentru care numărul 1·20 +2·21 +3·22 +. . .+n·2n−1
este pătrat perfect.
Stelian Corneliu Andronescu s, i Costel Bălcău, Pites, ti
Solut, ie. Fie a = 1 · + 2 · 21 + 3 · 22 + . . . + n · 2n−1 . Avem 2a = 1· 21 + 2 · 22 +
20
3 · 23 + . . . + n · 2n , deci 2a − a = n · 2n − 1 + 2 + 22 + . . . + 2n−1 . Rezultă că
a = n · 2n − (2n − 1) = (n − 1)2n + 1.
Pentru n = 1 avem a = 1 = p.p., pentru n = 2 avem a = 5 6= p.p., pentru
n = 3 avem a = 17 6= p.p., iar pentru n = 4 avem a = 49 = p.p. Fie acum n ≥ 5.
Avem a = p.p. ⇔ (n − 1)2n + 1 = (2k + 1)2 , k ∈ N ⇔ (n − 1)2n−2 = k(k + 1).
Cum k s, i k + 1 sunt prime ı̂ntre ele, rezultă că 2n−2 divide k sau 2n−2 divide
k + 1. Dacă 2n−2 divide k + 1, atunci k + 1 ≥ 2n−2 , deci k ≤ n − 1, prin urmare
2n−2 ≤ n, fals, deoarece utilizând inegalitatea 2m ≥ m + 1, pentru orice m ∗
m m−1 m−2
 ∈N
(ce poate fi dedusă astfel: 2 − 1 = (2 − 1) 2 +2 + . . . + 2 + 1 ≥ m)
avem 2 n−2 =2·2 n−3 ≥ 2(n − 2) > n. Dacă 2n−2 divide k, atunci k ≥ 2n−2 , deci
k + 1 ≤ n − 1, prin urmare 2 n−2 ≤ n − 2, din nou fals. În concluzie, singurele
solut, ii sunt n = 1 s, i n = 4.
Rezolvarea problemelor din numărul anterior 15

MGO 94. Fie ABCD un patrulater convex, T un punct ı̂n interiorul său, iar
M, N, P s, i Q mijloacele laturilor AB, BC, CD, respectiv DA. Dacă AAM T Q = a
s, i ACP T N = c, calculat, i aria patrulaterului ABCD ı̂n funct, ie de a s, i c.
Florea Badea, Scornices, ti
Solut, ie. Folosim proprietatea că mediana ı̂mparte triunghiul ı̂n două triunghiuri
de arii egale. Notând AT M A = AT M B = s1 , AT N B = AT N C = s2 , AT P C =
AT P D = s3 s, i AT QD = AT QA = s4 , avem AABCD = 2s1 + 2s2 + 2s3 + 2s4 =
2(s1 + s4 ) + 2(s2 + s3 ) = 2a + 2c.

MGO 95. Fie triunghiul ABC cu AB = AC s, i m (^A) = 100◦ . Se consideră


punctul D ı̂n semiplanul determinat de dreapta AB s, i care nu cont, ine punctul C,
astfel ı̂ncât m (^BCD) = 20◦ s, i BD = BC. Calculat, i măsura unghiului ADC.
Costel Anghel, Slatina
Solut, ie. Construim triunghiul echilateral EBC, cu punctul E de aceeas, i parte
a dreptei BC ca s, i punctul A. Atunci dreapta EA este mediatoarea lui [BC],
deci este s, i bisectoarea unghiului BEC, deci m (^AEB) = 30◦ . În triunghiul
isoscel CBD avem m (^CBD) = 180◦ − 2 · 20◦ = 140◦ , deci m (^ABD) =
m (^CBD) − m (^ABC) = 140◦ − 40◦ = 100◦ s, i m (^DBE) = m (^CBD) −
m (^ABE) = 140◦ − 60◦ = 80◦ . Dar BD = BC = BE. Rezultă că 4DBE este
180◦ − 80◦
isoscel, deci m (^BED) = = 50◦ s, i astfel m (^AED) = m (^AEB) +
2
m (^BED) = 30◦ + 50◦ = 80◦ . Din m (^ABD) + m (^AED) = 100◦ + 80◦ = 180◦ ,
rezultă că patrulaterul ABDE este inscriptibil, deci m (^ADB) = m (^AEB) =
30◦ , s, i astfel m (^ADC) = m (^ADB) − m (^BDC) = 30◦ − 20◦ = 10◦ .

Clasa a VIII-a

MGO 96. Fie a, b, c ∈ Q+ astfel ı̂ncât ab + bc + ca = a − b + 2. Demonstrat, i că


p
(a2 + 2a + 2)(b2 − 2b + 2)(c2 + 1) ∈ Q.
Costel Anghel, Slatina s, i Florea Badea, Scornices, ti
Solut, ie. Egalitatea dată poate fi scrisă sub forma (a+1)(b−1)+(b−1)c+c(a+1) = 1.
Notând S = (a + 1)(b − 1) + (b − 1)c + c(a + 1), avem a2 + 2a + 2 = (a + 1)2 + 1 =
(a + 1)2 + S = (a + b)(a + c + 1), b2 − 2b + 2 = (b − 1)2 + 1 = (b − 1)2 + S =
2 2
p + b)(b + c − 1) s, i c + 1 = c + S = (b + c − 1)(a + c + 1). Rezultă că
(a
2 2 2
(a + 2a + 2)(b − 2b + 2)(c + 1) = (a + b)(a + c + 1)|b + c − 1| ∈ Q.
16 Rezolvarea problemelor din numărul anterior
r
a2 + b2 + c2
MGO 97. Fie a, b, c > 0 cu a + b + c = 1. Arătat, i că + 18abc ≤ 1.
3
Marin Ionescu, Pites, ti
r
a2 + b2 + c2 a2 + b2 + c2
Solut, ie. Avem ≤ = a2 + b2 + c2 = (a + b + c)2 −
3  a + b+c
1 1 1 1 1 1 9
2(ab + ac + bc) = 1 − 2abc + + . Dar + + ≥ = 9, deci
r a b c a b c a + b + c
a2 + b2 + c2
≤ 1 − 18abc.
3
MGO 98. Arătat, i că pentru orice a, b, c > 0 are loc inegalitatea
a(a − b)(2b + c) b(b − c)(2c + a) c(c − a)(2a + b)
+ + ≥ 0.
b + 2c c + 2a a + 2b
Ardak Mirzakhmedov, Kazahstan
 
a(a − b)(2b + c) 2
Solut, ie. Inegalitatea dorită devine, succesiv: + a + 2ab +
   b + 2c
b(b − c)(2c + a) c(c − a)(2a + b)
+ b2 + 2bc + + c2 + 2ca ≥ (a + b + c)2 ⇔
c + 2a a + 2b
3a(ab + bc + ca) 3b(ab + bc + ca) 3c(ab + bc + ca)
+ + ≥ (a + b + c)2 ⇔
b + 2c c + 2a a + 2b
a b c (a + b + c)2
+ + ≥ . Ultima inegalitate se obt, ine, utilizând
b + 2c c + 2a a + 2b 3(ab + bc + ca)
a b c a2
Inegalitatea lui Titu Andreescu, astfel: + + = +
b + 2c c + 2a a + 2b ab + 2ca
b2 c2 (a + b + c) 2 (a + b + c)2
+ ≥ = .
bc + 2ab ca + 2bc (ab + 2ca) + (bc + 2ab) + (ca + 2bc) 3(ab + bc + ca)
MGO 99. Un trunchi de piramidă regulată este sect, ionat cu un plan paralel
cu bazele astfel ı̂ncât volumele celor două trunchiuri obt, inute să fie egale. Dacă
notăm cu L s, i l lungimile laturilor bazelor trunchiului dat s, i cu m lungimea laturii
poligonului de sect, iune, arătat, i că volumul cubului de muchie m este egal cu media
aritmetică a volumelor cuburilor de muchii L s, i l.
***
Solut, ie. Fie V vârful piramidei din care provine trunchiul. Notând cu v1 volumul
piramidei mici (cu vârful V s, i având ca bază baza mică a trunchiului) s, i cu v2
valoarea comună a volumelor celor două trunchiuri (obt, inute prin sect, ionarea
trunchiului dat), folosind rapoartele de asemănare dintre piramida mică, cea
mijlocie (cu vârful V s, i având ca bază poligonul de sect, iune) s, i cea mare (piramida
v1 l3 v1 l3
din care provine trunchiul), avem = 3 s, i = 3 . Rezultă că
v1 + v2 m v1 + 2v2 L
v1 l3 v1 l3 l3 2l3
= 3 s, i = 3 , deci 3 = 3 , de unde se obt, ine că
v2 m − l3 2v2 L − l3 m − l3 L − l3
L3 + l 3
m3 = , ceea ce trebuia demonstrat.
2
Rezolvarea problemelor din numărul anterior 17

MGO 100. Fie a > 0 un număr real fixat. Determinat, i valorile x ∈ R pentru care
există două piramide V ABCD s, i V A0 B 0 C 0 D0 astfel ı̂ncât ABCD este un pătrat,
A0 B 0 C 0 D0 este un romb, AB = A0 B 0 = a, m (^B 0 A0 D0 ) = 60◦ , triunghiurile V AB
s, i V A0 B 0 sunt echilaterale, iar perimetrele triunghiurilor V CD s, i V C 0 D0 sunt
egale fiecare cu x.

Stelian Corneliu Andronescu s, i Costel Bălcău, Pites, ti

Solut, ie. Fie M mijlocul lui [AB] s, i N mijlocul lui [CD]. Punctul V apart, ine
planului mediator al lui [AB], care este s, i planul mediator al lui [CD], deci
V N ⊥ CD. Fie t = m (^V M N ). Conform Teoremei cosinusului avem V N 2 =
3a2 √
V M 2 + M N 2 − 2V M · M N cos t = + a2 − a2 3 cos t. Aplicând Teorema lui
r4
a2 p √
Pitagora obt, inem că V C = V D = V N 2 + = a 2 − 3 cos t, prin urmare
4


 q 
x = P4V CD = a 1 + 2 2 − 3 cos t .

Cum t ia toate valorile din intervalul (0◦ , 180◦ ), deci cos t ia toate valorile din
intervalul (−1, 1), deducem că x = P4V CD ia toate valorile din intervalul
!
√ √
 q   q
a 1 + 2 2 − 3 ,a 1 + 2 2 + 3
 √ √  √ √ 
= a 1 + 6 − 2 ,a 1 + 6 + 2 , (1).

Pe de altă parte, punctele V s, i D0 apart, in planului mediator al lui [A0 B 0 ],


deci avem V D0 ⊥ A0 B 0 , prin urmare avem s, i V D0 ⊥ C 0 D0 . Fie t0 = m (^V M D0 ).
Conform Teoremei cosinusului avem V D√0 2 = V M 2 + M D0 2 − 2V M · M D0 cos t0 =
3a2 3a2 a 3 √
2· −2· · cos t0 , deci V D0 = · 2 − 2 cos t0 . Aplicând Teorema lui
4 4 2
p a √
Pitagora obt, inem că V C 0 = V D0 2 + a2 = · 10 − 6 cos t0 , prin urmare
2
a √ √ 
x = P4V C 0 D0 = 2 + 6 − 6 cos t0 + 10 − 6 cos t0 .
2

Cum t0 ia toate valorile din intervalul (0◦ , 180◦ ), deci cos t0 ia toate valorile din
intervalul (−1, 1), deducem că x = P4V C 0 D0 ia toate valorile din intervalul
 √ 
2a, a 3 + 3 , (2).

 √ √  √ 
Din (1) s, i (2), prin intersect, ie rezultă că x ∈ a 1 + 6 − 2 , a 3 + 3 .
18 Rezolvarea problemelor din numărul anterior

Clasa a IX-a

MGO 101. Pentru orice numere reale pozitive a1 , a2 , . . . , an s, i orice număr


natural k mai mare ca 1 are loc inegalitatea

(ak1 + n − 1)(ak2 + n − 1) · . . . · (akn + n − 1) ≥ nn−k (a1 + a2 + . . . + an )k .

Ardak Mirzakhmedov, Kazahstan

Solut, ie. Presupunem, fără a restrânge generalitatea. că a1 ≤ a2 ≤ . . . ≤ am ≤ 1 ≤


am+1 ≤ . . . ≤ an , cu m ∈ {0, 1, . . . , n}. ConformInegalităt, ii generalizate a lui Ber-
m k m k P aki − 1
m

Q ai + n − 1 a −1
1+ i
Q
noulli, pentru m ≥ 1 avem = ≥1+ =
m i=1 n i=1 n i=1 n
m m
  
1 P k
aki + n − 1 ≥ nm−1 aki + n − m , (1), iar
Q  P
ai + n − m , deci
n i=1 i=1 i=1 
n aki + n − 1 n aki − 1 n aki − 1

Q Q P
pentru m < n avem = 1+ ≥ 1+ =
n n n
 n  i=m+1 n i=m+1  n i=m+1 
1 P k
aki + n − 1 ≥ nn−m−1
Q  P k
ai + m , deci ai + m , (2).
n i=m+1 i=m+1 i=m+1
Din (1) s, i (2) prin ı̂nmult, ire rezultă că

n  m n
! !
Y  X X
aki + n − 1 ≥ nn−2 aki + n − m aki + m , (3),
i=1 i=1 i=m+1

inegalitate valabilă s, i pentru m = 0 sau m = n. Scriind nk−2 = (1 + 1 + . . . + 1)k−2 ,


conform Inegalităt, ii generalizate a lui Hölder avem

m
! n
! n
!k
X X X
nk−2 aki + n − m m+ aki ≥ ai , (4).
i=1 i=m+1 i=1

Din (3) s, i (4) rezultă inegalitatea din enunt, .

MGO 102. Să se determine valoarea minimă a expresiei

2(x3 + y 3 + z 3 ) + 3xyz
E(x, y, z) =
x2 y + xy 2 + y 2 z + yz 2 + z 2 x + zx2

când x, y, z ∈ (0, ∞).

Dan Nedeianu, Drobeta Turnu Severin s, i Sorin Ulmeanu, Pites, ti


Rezolvarea problemelor din numărul anterior 19

3
Solut, ie. Evident, E(x, x, x) = . Arătăm că pentru orice x, y, z > 0 avem
2
3
E(x, y, z) ≥ , inegalitate echivalentă cu 4 x3 ≥ 3 xy(x+y)−6xyz, (1). Dar,
P P
2 P 3 P
conform Inegalităt, ii lui Schur, avem
P 4 x ≥ 4 + y) − 12xyz, (2), deci
xy(x P
este suficient
P să demonstrăm că 4 xy(x+y)−12xyz ≥ 3 xy(x+y)−6xyz, (3),
adică xy(x + y) ≥ 6xyz, inegalitate care rezultă imediat folosind Inegalitatea
3
mediilor. Din (2) s, i (3) rezultă (1), deci min E(x, y, z) = .
x,y,z>0 2
MGO 103. Rezolvat, i ı̂n R × R ecuat, ia

(sin3 x + 2)(cos3 y − 3 sin2 y − 3) + (cos y − 3)(2 cos2 x − sin x − 2) = 0.


Mihai Florea Dumitrescu, Potcoava
2 sin2 x + sin x cos3 y + 3 cos2 y − 6
Solut, ie. Ecuat, ia se poate rescrie sub forma = ,
sin3 x + 2 cos y − 3
2t2 + t
adică f (sin x) = g(cos x), considerând funct, iile f, g : [−1, 1] → R, f (t) = 3 s, i
t +2
3 2
t + 3t − 6 2
(t − 1)(2 − t)
g(t) = . Pentru orice t ∈ [−1, 1] avem f (t) − 1 = ≤0
t−3 t3 + 2
(t2 − 1)(t + 3)
s, i g(t) − 1 = ≥ 0, deci f (t) ≤ 1 ≤ g(t), iar fiecare egalitate are
t−3
loc dacă s, i numai dacă t = ±1. Prin urmare sin x = ±1 s, i cos y = ±1, adică
π
x = + kπ s, i y = mπ, cu k, m ∈ Z.
2
MGO 104. Fie ABC un triunghi neechilateral ı̂nscris ı̂ntr-un cerc de centru O
s, i circumscris unui cerc de centru I. Notăm cu M , N s, i P centrele cercurilor
exı̂nscrise triunghiului ABC. Să se arate că centrul de greutate al triunghiului
M N P se află pe dreapta OI.
Marin Ionescu, Pites, ti
Solut, ie. Fie D, E s, i F punctele de intersect, ie ale segmentelor [IM ], [IN ], respectiv
[IP ] cu cercul circumscris 4ABC, deci D ∈ AI, E ∈ BI, F ∈ CP . Triunghiul
IBM este dreptunghic ı̂n B (deoarece BI s, i BM sunt bisectoarea interioară,
respectiv exterioară ale unghiului ABC) s, i BD = ID (deoarece m (^BID) =
_ _ _ _ _
m(BD) + m(AE) m (^BAC) + m (^ABC) m(CD) + m(CE) m(DE)
= = = =
2 2 2 2
m (^IBD)), deci D este mijlocul lui [IM ]. Analog, E este mijlocul lui [IN ] s, i
−−→ −→ −−→ −−→ −−→ −→
F este mijlocul lui [IP ]. Rezultă că 2OD = OI + OM , deci OM = 2OD − OI
−−→ −−→ −→ −−→ −−→ −→
s, i, analog, ON = 2OE − OI, OP = 2OF − OI. Notând cu V centrul de greu-
tate al triunghiului M N P , utilizând Relat,ia lui Leibniz s, i egalităt
, ile anterioare
−−→ −−→ −−→ −−→ −−→ −−→ −−→ −→
rezultă că 3OV = OM + ON + OP = 2 OD + OE + OF − 3OI, (1). Dar
I este ortocentrul triunghiului DEF (deoarece, de exemplu, m (^(DA, EF )) =
_ _ _ _ _
m(AF ) + m(DE) m(AF ) + m(DC) + m(CE) 1
= = m (^ACB)+m (^BAC)+
2 2 2
20 Rezolvarea problemelor din numărul anterior

m (^ABC) = 90◦ ), deci conform Relat, iei lui Sylvester pentru 4DEF avem
−→ −−→ −−→ −−→ −−→ −→
OI = OD + OE + OF , (2). Din (1) s, i (2) rezultă că 3OV = −OI, deci punctele
O, V s, i I sunt coliniare.
Y ra X ra
MGO 105. Arătat, i că ı̂n orice triunghi ABC avem 4 · − = 1.
ha ha
Daniel Jinga, Pites, ti
ra S a a
Solut, ie (Daniel Văcaru, Pites, ti). Avem = · = . Rezultă că
ha p − a 2S 2(p − a)
Q ra abc pabc 2R P ra a
4· = = 2
= , (1) s, i = +
ha 2(p − a)(p − b)(p − c) 2S r ha 2(p − a)
b c p2 (a + b + c) − 2p(ab + bc + ca) + 3abc
+ = . Dar ab + bc + ca =
2(p − b) 2(p − c) 2(p − a)(p − b)(p − c)
P ra pr(2R − r)
p2 + r2 + 4Rr s, i abc = 4RS = 4pRr, deci = =
ha (p − a)(p − b)(p − c)
p2 r(2R − r) 2R − r
= , (2). Din (1) s, i (2) obt, inem relat, ia din enunt, .
S2 r

Clasa a X-a

MGO 106. Fie x, y, z ∈ (0, ∞) astfel ı̂ncât x + y + z = xyz. Demonstrat, i că



x y z 3 3
+ + ≤ .
1 + x2 1 + y 2 1 + z 2 4

Când are loc egalitatea?


Alexandru Daniel Pı̂rvuceanu, elev, Drobeta Turnu Severin
Solut, ie (Daniel Văcaru, Pites, ti). Din condit, iile date rezultă că există un triunghi
ascut, itunghic ABC astfel ı̂ncât x = tg A, y = tg B s, i z = tg C. Astfel avem
x tg A sin 2A y
2
= 2 = tg A cos2 A = sin A cos A = s, i, analog, =
1+x 1 + tg A 2 1 + y2
sin 2B z sin 2C x y z sin 2A + sin 2B + sin 2C
, 2
= , deci 2
+ 2
+ 2
= .
2 1+z 2 1+x 1+y 1+z 2
Cum funct, ia sin este concavă pe intervalul (0, π), conform Inegalităt, ii lui √ Jensen
sin 2A + sin 2B + sin 2C 3 2(A + B + C) 3 2π 3 3
avem ≤ · sin = · sin = , de
2 2 3 2 3 4
unde obt, inem concluzia.
π √
Egalitatea are loc dacă s, i numai dacă A = B = C = , adică x = y = z = 3.
3
Rezolvarea problemelor din numărul anterior 21

MGO 107. Fie a > 0, a 6= 1. Rezolvat, i ı̂n mult, imea numerelor reale ecuat, ia

loga (x − 2a − 1) = log2a+1 (2x + 2a2 − 1).


Marin Chirciu, Pites, ti
Solut, ie. Domeniul de definit, ie al ecuat, iei date este D = (2a + 1, ∞). Notând
loga (x − 2a − 1) = log2a+1 (2x + 2a2 − 1) = t, rezultă că x − 2a − 1 = at s, i
2x + 2a2 − 1 = (2a + 1)t , deci 2x = 2at + 4a + 2 = (2a + 1)t − 2a2 + 1. Împărt, ind
 t  t
a 1
prin (2a + 1)t obt, inem că 2 + 2a2 + 4a + 1

= 1. Aceasta
2a + 1 2a + 1
este o ecuat, ie de forma f (t) = 1 cu f strict descrescătoare, având solut, ia unică
t = 2. Rezultă că x = (a + 1)2 ∈ D este solut, ie unică pentru ecuat, ia dată.
MGO 108. Fie numerele reale a, b s, i c astfel ı̂ncât

th (a)th (b) + th (b)th (c) + th (c)th (a) = 1

ex − e−x
(unde th (x) = reprezintă tangenta hiperbolică a numărului real x).
ex + e−x
a) Demonstrat, i că a, b, c < 0 sau a, b, c > 0.

1 2 + th (a) + th (b) + th (c) + th (a)th (b)th (c)
b) Arătat, i că a + b + c = ln .
2 2 − th (a) + th (b) + th (c) + th (a)th (b)th (c)
Michel Bataille, Frant, a s, i Leonard Mihai Giugiuc, România
Solut, ie. Evident, −1 < th (x) < 1, pentru orice x ∈ R. De asemenea, avem
echivalent, ele: th (x) = 0 ⇔ x = 0; th (x) > 0 ⇔ x > 0; th (x) < 0 ⇔ x < 0.
Notăm th (a) = u, th (b) = v s, i th (c) = w. Atunci u, v, w ∈ (−1, 1) s, i
uv + vw + wu = 1.
a) Presupunem prin reducere la absurd că numerele a, b, c nu au toate acelas, i
semn. Atunci ar exista două dintre ele, să zicem a s, i b, astfel ı̂ncât a ≥ 0 ≥ b. Prin
urmare u ≥ 0 ≥ v. Dacă w ≥ 0 ar rezulta că 1 = uv + vw + wu ≤ wu < 1, fals, iar
dacă w < 0 ar rezulta că 1 = uv + vw + wu ≤ vw < 1, din nou fals.
1 1+u 1 1+v
b) Din th (a) = u rezultă că a = ln . Analog, avem b = ln
2 1−u 2 1−v
1 1+w 1 (1 + u)(1 + v)(1 + w)
s, i c = ln . Astfel obt, inem că a + b + c = ln =
2 1−v 2 (1 − u)(1 − v)(1 − w)
1 2 + (u + v + w + uvw)
ln , ceea ce trebuia demonstrat.
2 2 − (u + v + w + uvw)
MGO 109. Fie z1 , z2 , z3 ∈ C∗ astfel ı̂ncât |z1 +z2 | = |z1 |+|z2 |, |z1 +z3 | = |z1 |+|z3 |
s, i z2 + z3 = 2z1 . Determinat, i mult, imea tuturor valorilor posibile pentru numărul
z 2 · z2 + z3 · z3
.
z1 · z 1
Mihai Florea Dumitrescu, Potcoava
22 Rezolvarea problemelor din numărul anterior

Solut, ie. Cum z1 , z2 , z3 ∈ C∗ , egalitatea |z1 + z2 | = |z1 | + |z2 | are loc dacă s, i numai
dacă există k ∈ R∗+ astfel ı̂ncât z2 = kz1 , iar egalitatea |z1 + z3 | = |z1 | + |z3 | are
loc dacă s, i numai dacă există p ∈ R∗+ astfel ı̂ncât z3 = pz1 . Dar z2 + z3 = 2z1 ,
z2 · z2 + z3 · z3
deci k + p = 2, adică p = 2 − k, cu k ∈ (0, 2). Astfel obt, inem =
z1 · z 1
|z2 |2 + |z3 |2
= k 2 + p2 = k 2 + (2 − k)2 = 2k 2 − 4k + 4 = 2(k − 1)2 + 2. Avem
|z1 |2
echivalent, ele k ∈ (0, 2) ⇔ k−1 ∈ (−1, 1) ⇔ (k−1)2 ∈ [0, 1) ⇔ 2(k−1)2 +2 ∈ [2, 4),
deci mult, imea cerută este intervalul [2, 4).

MGO 110. Rezolvat, i ı̂n mult, imea numerelor reale ecuat, ia

cos2 x + cos2 2x · sin2 3x + sin2 4x = 1.

Daniel Jinga, Pites, ti

Solut, ia 1. Ecuat, ia dată devine, succesiv: cos2 2x · sin2 3x + sin2 4x −h sin2 x = 0;


2
cos2 2x·sin2 x 3 − 4 sin2 x +16 sin2 x·cos2 x·cos2 2x−sin2 x = 0; sin2 x cos2 2x(1+
i
2 cos 2x)2 + 8 cos2 2x(1 + cos 2x) − 1 = 0. Cazul 1. sin2 x = 0, cu solut, ia x = kπ,
k ∈ Z. Cazul 2. t2 (1 + 2t)2 + 8t2 (1 + t) − 1 = 0, unde t = cos 2x ∈ [−1, 1].
2
Efectuând calculele, ecuat, ia devine 4t4 + 12t3 + 9t2 − 1 = 0, adică 2t2 + 3t − 1 =
1
0, adică 2t2 + 3t + 1 2t2 + 3t − 1 = 0, având solut, iile t1 = −1, t2 = − ,
 
√ √ 2
−3 + 17 −3 − 17
t3 = ∈ [−1, 1], t4 = 6∈ [−1, 1]. Astfel mult, imea solut, iilor
4 4n
π o n π o
ecuat, iei date este S = {kπ | k ∈ Z} ∪ + kπ k ∈ Z ∪ ± + kπ k ∈ Z ∪
( √ ) 2 3
1 17 − 3
± arccos + kπ k ∈ Z .
2 4

Solut, ia 2 (Daniel Văcaru, Pites, ti). Putem scrie, echivalent, cos2 2x · sin2 3x =
sin2 x − sin2 4x, adică cos2 2x · sin2 3x = − sin 3x · sin 5x, prin urmare sin 3x = 0 sau
cos2 2x · sin 3x = − sin 5x. Pentru a doua ecuat, ie, cum sin 3x 6= 0, obt, inem, suc-
sin 5x sin 3x + sin 5x 2 sin 4x cos x
cesiv: cos2 2x = − ; sin2 2x = ; 4 sin2 x cos2 x = ,
sin 3x sin 3x sin 3x
sin 4x
deci cos x = 0 sau 2 sin2 x cos x = . Ultima ecuat, ie devine, succesiv:
sin 3x
sin x sin 2x sin 3x = sin 4x; sin x sin 2x sin 3x = 2 sin 2x cos 2x, cu posibilităt, ile
sin 2x = 0 sau sin x sin 3x = r2 cos√ 2x, echivalentă cu 4 sin4 x − 7 sin2 x + 2 = 0, cu
7 − 17
solut, ia posibilă sin x = ± . În concluzie, mult, imea solut, iilor ecuat, iei
8
date este r reuniunea solut, iilor ecuat, iilor sin 3x = 0, cos x = 0, sin 2x = 0 s, i

7 − 17
sin x = ± . Rezolvarea acestora conduce la aceeas, i mult, ime S ca la
8
solut, ia anterioară.
Rezolvarea problemelor din numărul anterior 23

Clasa a XI-a

MGO 111. Fie A ∈ Mn (C) astfel ı̂ncât A6 = A + In . Demonstrat, i că matricea


A2 + A + In este inversabilă.
Cristinel Mortici, Viforâta
Solut, ie (Daniel Văcaru, Pites, ti). Avem A(A5 − In ) = In , deci det A 6= 0. Din
A = A6 − In = (A3 − In )(A3 + In ) = (A − In )(A2 + A + In )(A + In )(A2 − A + In )
s, i det A 6= 0 rezultă că det(A2 + A + In ) 6= 0, deci matricea A2 + A + In este
inversabilă (mai mult, se deduce că toate matricele A, A5 − In , A3 − In , A3 + In ,
A − In , A2 + A + In , A + In , A2 − A + In sunt inversabile).
MGO 112. Fie a, b ∈ C∗ astfel ı̂ncât a 6= ±b s, i A, B ∈ M4 (C) astfel ı̂ncât
det(aAB + bBA) = det(aBA + bAB). Demonstrat, i că
 
det (x + a)AB + (b − x)BA = det (x + a)BA + (b − x)AB , ∀x ∈ C.
Daniel Jinga, Pites, ti

 f : C → C, f (x) = det (x + a)AB
Solut, ie. Fie  + (b − x)BA − det (x + a)BA + 
(b − x)AB = det aAB + bBA + x(AB − BA) − det aBA + bAB + x(BA − AB) .
Deoarece det(AB − BA) = det(BA − AB) s, i det(aAB + bBA) = det(aBA + bAB)
3 2
 forma f (x) = αx +βx +γx, cu α, β, γ ∈ C. Evident, avem f (0) =
rezultăcă f are
b−a  
0 s, i f = 0. De asemenea, f (b) = det (b + a)AB − det (b + a)BA = 0
2
  b−a
s, i f (−a) = det (b + a)BA − det (b + a)AB = 0. Cum numerele 0, , b s, i
2
−a sunt distincte două câte două, iar f este funct, ie polinomială de grad cel mult
3, rezultă că f = 0 s, i problema este rezolvată.
MGO 113. Se consideră matricea A ∈ Mn (C), n ≥ 2, astfel ı̂ncât det A = 0 s, i
det(In + A∗ ) = 1, unde A∗ este matricea adjunctă a lui A. Arătat, i că (A∗ )2 = On .
Marin Ionescu, Pites, ti
Solut, ie. Deoarece det A = 0, avem rang A ≤ n − 1. Dacă rang A = n − 2, atunci
A∗ = On , deci s, i (A∗ )2 = On . Fie acum rang A = n − 1. Avem AA∗ = det A · In =
On , deci din Inegalitatea lui Sylvester rang A + rang A∗ ≤ n 
+ rang (AA∗ ) obt, inem

b1 0 . . . 0
 b2 0 . . . 0 
rang A∗ ≤ 1. Rezultă că A∗ are forma A∗ = BC, cu B =  .  s, i
 
 .. 
bn 0 . . . 0
 
c1 c2 . . . cn
 0 0 ... 0 
C= , unde b1 , b2 , . . . , bn , c1 , c2 , . . . , cn ∈ C. Avem tr A∗ = t,
 ... 
0 0 ... 0
24 Rezolvarea problemelor din numărul anterior
 
1 0 ... 0
Pn  0 0 ... 0 
unde t = bk ck . Avem s, i CB = tE, unde E =   ∈ Mn (C).
 ... 
k=1
0 0 ... 0
∗ 2 ∗
Rezultă că (A ) = BCBC = tBEC = tBC = tA . Dar, folosind ipoteza s, i
Identitatea lui Sylvester avem 1 = det(In + A∗ ) = det(In + BC) = det(In + CB),
deci 1 = det(In + tE) = 1 + t. Rezultă că t = 0 s, i astfel (A∗ )2 = 0 · A∗ = On .

MGO 114. Fie (an )n≥1 o progresie aritmetică având primul termen a1 = 1 s, i
1 1 1
a a
an1 + an2 + . . . + anan
rat, ia r ∈ N∗ . Calculat, i lim .
n→∞ an
Dan S, tefan Marinescu, Hunedoara s, i Leonard Mihai Giugiuc, Drobeta Turnu Severin

r+1
Solut, ie. Vom arăta că limita este . Pentru aceasta, fie (xn )n≥1 s, irul căruia
r
an + n − 1
trebuie să-i aflăm limita. În mod cert avem ≤ xn , ∀n ∈ N∗ , (1).
an s
1
ak 2(an − 1)
Pe de altă parte, pentru orice n ≥ k ≥ 2 avem an < 1 + , (2).
ak (ak − 1)
s !ak
2(an − 1)
Într-adevăr, utilizând Binomul lui Newton obt, inem 1 + >
ak (ak − 1)
2(an − 1)
1 + Ca2k · = 1 + an − 1 = an . Din (2) suntem condus, i la
ak (ak − 1)
s
n
an + n − 1 2(an − 1) 1 X 1
xn < + ·√ · p , ∀n ≥ 2 (3).
an an an ak (ak − 1)
k=2

an + n − 1 r+1 1 n
P 1
Cum lim = s, i lim √ · p = 0 (cu Lema Stolz-
n→∞ an r n→∞ an k=2 ak (ak − 1)
1
p √ √
an+1 (an+1 − 1) 1 an+1 + an
Cesaro ea este egală cu lim √ √ = · lim p = 0),
n→∞ an+1 − an r n→∞ an+1 (an+1 − 1)
r+1
din (1) s, i (3) conform Criteriului cles, telui deducem că lim xn = .
n→∞ r
MGO 115. a) Fie a, b, c, d ≥ 0 cu a2 + b2 + c2 + d2 = 2. Demonstrat, i că

a + b + c + d − (abc + abd + acd + bcd) ≤ 2.

b) Arătat, i că pentru orice k < 1 există a, b, c, d ≥ 0 cu a2 + b2 + c2 + d2 = 2 astfel


ı̂ncât a + b + c + d − k(abc + abd + acd + bcd) > 2.

Leonard Mihai Giugiuc, Drobeta Turnu Severin


Rezolvarea problemelor din numărul anterior 25

Solut, ia 1. Vom utiliza următoarele două rezultate.


Lema 1 (problema MGO 79 din RMGO 1/2018). Fie α > β > 0 s, i a, b, c, d ≥ 0
astfel ı̂ncât a + b + c + d = 2α + β s, i a2 + b2 + c2 + d2 = 2α2 + β 2 . Atunci
abc + abd + acd + bcd ≥ α2 β, cu egalitate pentru (α, α, β, 0) s, i permutările sale.
Lema 2 (problema L 355 din RecMat 2/2018). Fie α ≥ β ≥ 0 s, i a, b, c, d ≥ 0
astfel ı̂ncât a + b + c + d = 3α + β s, i a2 + b2 + c2 + d2 = 3α2 + β 2 . Atunci
abc + abd + acd + bcd ≥ α3 + 3α2 β, cu egalitate pentru (α, α, β, α) s, i permutările
sale.
a) Avem (a + b + c + d)2√≥ a2 + b2 + c2 + d2 √ = 2 s, i (a + b + c + d)2 ≤
4(a2 + b2 + c2 + d2 ) = 8, deci 2 ≤ a + b + c + d ≤ 2 2.

Cazul 1. 2 ≤ a + b + c + d ≤ 2. Atunci a + b + c + d − (abc + abd + acd + bcd) ≤
a + b + c + d = 2, cu egalitate pentru (1, 1, 0, 0) s, i permutările sale.
√ !
√ 2 √ 
Cazul 2. 2 < a + b + c + d < 6. Funct, ia f : √ , 1 → 2, 6 , f (x) =
3
√ !
√ 2
2x + 2 − 2x2 este bijectivă, deci există x ∈ √ , 1 astfel ı̂ncât a + b + c + d =
3
√ √
2x + 2 − 2x . Aplicând√Lema 1 pentru α = x s, i β = 2 − 2x2 rezultă că
2

abc +√ abd + acd + bcd √ ≥ x2 2 − 2x2 , deci a + b + c + d − (abc + abd + acd √ + bcd) ≤
2 2 2
+ 2 − 2x − x 2 − 2x . Astfel este 2
2x√
2
√ suficient √ să arătăm că 2x + 2 − 2x −
2 2
x 2 − 2x < 2, echivalent cu (x + 1) 1 − x < 2, care este adevărată deoarece
√ 1 1
x + 1 < 2 s, i 1 − x2 ≤ √ < √ .
3 2
" √ #
√ √ 1 2 √ √ 
Cazul 3. 6 ≤ a + b + c + d ≤ 2 2. Funct, ia g : √ , √ → 6, 2 2 ,
2 3
" √ #
√ 1 2
g(x) = 3x + 2 − 3x2 este bijectivă, deci există x ∈ √ , √ astfel ı̂ncât
2 3
√ √
a + b + c + d = 3x + 2 − 3x2 . Aplicând Lema √ 2 pentru α = x s, i β = 2 − 3x2
3 2 2
rezultă că abc + abd + acd√+ bcd ≥ x + 3x 2√− 3x , deci a + b + c + d − (abc +
abd + acd + bcd)√≤ 3x + 2 − 3x2 −√x3 − 3x2 2 − 3x2 . Astfel este suficient să
2 3 2 2
√ 3x + 2 − 3x − x −3 3x 2 − 3x < 22, care este adevărată deoarece
arătăm că
2 2
(1 − 3x ) 2 − 3x ≤ 0 s, i 3x − x − 2 = −(1 − x) (x + 2) < 0.
√ 
2
b) Fie k < 1. Din demonstrat, ia Cazului 2, pentru orice x ∈ √ , 1
√ √ 3
2x + 2 − 2x2 − 2 2x + 2 − 2x2 − 2 1
avem √ < 1. Deoarece lim √ = lim 2

√ r x 2 − 2x
2 2 x%1 2
x 2 − 2x √
2 x%1 x
√
2x + 2 − 2x2 − 2

2 1−x 2
· = 1, există x ∈ √ , 1 astfel ı̂ncât √ > k, de
x2 1+x 3 x2 2 − 2x2
unde, conform demonstrat, iei Cazului 2 s, i cazului de egalitate din Lema 1, deducem
26 Rezolvarea problemelor din numărul anterior

că există a, b, c, d ≥ 0 cu a2 + b2 + c2 + d2 = 2 astfel ı̂ncât a + b + c + d − k(abc +


abd + acd + bcd) > 2.
Solut, ia 2 pentru punctul a) (Thuan Nguyen Hoang, Vietnam). Pentru a + b + c +
d − (abc + abd + acd + bcd) ≤ 0 concluzia este evidentă. Fie acum a + b + c + d −
(abc + abd + acd + bcd) > 0. Inegalitatea dorită h este echivalentă cu f (a, b, c, d) ≥ 0,
unde f (a, b, c, d) = 2(a2 + b2 + c2 + d2 )3 − (a + b + c + d)(a2 + b2 + c2 + d2 ) −
i2
2(abc + abd + acd + bcd) . Putem presupune, fără a restrânge generalitatea, că
d = min{a, b, c, d}. Pentru (a, b, c, d) fixat, notăm F (t) = f (a+t, b+t, c+t, d+t), cu
t ≥ −d. Avem F 0 (t) = 24AB, unde A = a2 +b2 +c2 +d2 +2(a+b+c+d)t+4t2 ≥ 0,
B = abc+abd+acd+bcd+2(ab+ac+bc+ad+bd+cd)t+3(a+b+c+d)t2 +4t3 ≥ 0.
Astfel F 0 (t) ≥ 0, deci F (t) ≥ F (−d), pentru orice t ≥ −d. Astfel F (0) ≥ F (−d),
adică f (a, b, c, d) ≥ f (a−d, b−d, c−d, 0). Prin urmare, este suficient să demonstrăm
inegalitatea dorită doar ı̂n cazul d = 0, adică a + b + c − abc ≤ 2, pentru
a2 + b2 + c2 = 2 s, i a, b, c ≥ 0. Utilizând Metoda uvw, este suficient să arătăm
această relat, ie doarp ı̂n următoarele două cazuri. Cazul 1. c = 0. Atunci avem de
arătat că√a + b ≤ 2(a 2 2
2
√ + b ), adevărat. Cazul 2. a = b. Atunci avem de arătat
că 2a + 2 − 2a − a 2 − 2a2 ≤ 2 pentru 0 ≤ a ≤ 1, relat, ie us, or de demonstrat.
2

Clasa a XII-a

MGO 116. Fie a, b ∈ Z∗23 . Calculat, i probabilitatea ca alegând un endomorfism f


al grupului (Z∗23 , ·) acesta să verifice relat, ia f (a) = b.
Stelian Corneliu Andronescu s, i Costel Bălcău, Pites, ti
Solut, ie. Avem Z∗23 = h b 5 i, deci orice endomorfism f este bine determinat de
elementul f (b 5m ∈ Z∗23 , m = 0, 21. Rezultă că avem 22 de endomorfisme,
5) = b
având forma f b 5k = b 5km pentru orice k = 0, 21, unde m = 0, 21. Fie a = b
5p s, i
 
b=b 5q , cu p, q = 0, 21. Avem echivalent, ele: f (a) = b ⇔ f b
5p = b 5q ⇔ b
5pm = b
5q
⇔ pm ≡ q (mod 22), (1). Avem următoarele cazuri.
Cazul 1. p 6∈ {0, 2, 11}, adică a 6∈ {b1, b
2, 22}.
b Atunci ecuat, ia (1) are solut, ie m
1
unică pentru orice q, deci probabilitatea cerută este P = , pentru orice b.
22
Cazul 2. p = 0, adică a = 1. Subcazul 2.1. q = 0, adică b = b
b 1. Atunci ecuat, ia
22
(1) are ca solut, ie orice m = 0, 21, deci P = 6 0, adică
= 1. Subcazul 2.2. q =
22
b 6= b
1. Atunci ecuat, ia (1) nu are solut, ii, deci P = 0.
2. Atunci ecuat, ia (1) devine 2m ≡ q (mod 22).
Cazul 3. p = 2, adică a = b
Subcazul 3.1. q = 2l, cu l = 0, 10, adică b ∈ {b 1, b
2, b
4, b
8, 16,
b b 9, 18,
b 13, 3, b
b b 6, 12}.
b
Rezolvarea problemelor din numărul anterior 27

Atunci ecuat, ia (1) devine 2m ≡ 2l (mod 22), deci are solut, iile m ∈ {l, l + 11},
2 1
deci P = = . Subcazul 3.2. q = 2l + 1, cu l = 0, 10, adică b ∈
22 11
{b
5, 10,
b 20,
b 17,
b 11,b 22,b 21,
b 19,
b 15,
b b 7, 14}.
b Atunci (1) devine 2m ≡ 2l + 1 (mod 22),
deci nu are solut, ii, deci P = 0.
Cazul 4. p = 11, adică a = 22 b = −b 1. Atunci (1) devine 11m ≡ q (mod 22).
Subcazul 4.1. q 6∈ {0, 11}, adică b 6∈ {b
1, 22}.
b Atunci (1) nu are solut, ii, deci P = 0.
Subcazul 4.2. q = 0, adică b = b 1. Atunci (1) are solut, iile m = 2l, cu l = 0, 10,
11 1
deci P = = . Subcazul 4.3. q = 11, adică b = 22. b Atunci (1) are solut, iile
22 2
11 1
m = 2l + 1, cu l = 0, 10, deci din nou P = = .
22 2
MGO 117. Fie p un număr prim de forma 4k + 1. Demonstrat, i că polinomul
f = X p−1 + aX + b
1 ∈ Zp [X] este reductibil, pentru orice a ∈ Zp .
Rămâne afirmat, ia adevărată pentru numerele prime p de forma 4k + 3?
***
Solut, ie. Cazul 1. a = b 0. Atunci f = X p−1 +b
1. Cum p = 4k+1, există m ∈ Zp astfel
2 b [
ı̂ncât m = −1 (ı̂ntr-adevăr, pentru m = (2k)! folosind că x b = −p[
− x pentru orice
x ∈ {1, 2, . . . , 2k} s, i Teorema lui Wilson 2 2k [ \
avem m = (−1) · (4k)! = (p − 1)! = −1).
b
4k 2 2k 2k

Astfel f = X − m = X − m X + m , deci f este reductibil.
Cazul 2. a 6= b 0. Atunci pentru orice x ∈ Z∗p avem xp−1 = b
1 (Mica teoremă a
 
lui Fermat), deci f (x) = ax + b 2. Prin urmare f −b2·a−1 0, deci f se divide
=b
cu X + b2 · a−1 s, i astfel este reductibil.
b) Afirmat, ia nu rămâne neapărat adevărată pentru p = 4k + 3. De exemplu, ı̂n
Z3 polinomul f = X 2 + b 1 este ireductibil, deoarece f (x) 6= b
0 pentru orice x ∈ Z3 .

 a+b+c+d=4
4
MGO 118. Rezolvat, i ı̂n R sistemul a2 + b2 + c2 + d2 = 52 .
 4
a + b4 + c4 + d4 = 868
Florentin Vis, escu, Bucures, ti s, i Leonard Mihai Giugiuc, Drobeta Turnu Severin
Solut, ia 1. Fie x4 −px 3
P +qx
2 −rx+t = 0 ecuatia având rădăcinile a, b, c si d. Atunci
, ,
2
p − a 2 16 − 52 P 3 P 2 P
p = 4 s, i q = = = −18. Cum a − p a + q a − 4r +
2 2
P1 r
= 0, obt, inem a = 4·52+18·4+4r −t· = 280+3r. Dar a4 −p a3 +
P 3 P P
t
Pa t
q a2 −r a+4t = 0, adică 868−4(280+3r)−18·52−4r+4t = 0, deci t = 4r+297.
P
Ecuat, ia devine x4 −4x3 −18x2 −rx+4r+297 = 0. Cum x = 4 nu este solut, ie, ecuat, ia
x4 − 4x3 − 18x2 + 297 9
devine −r = 0, sau, echivalent, x3 −18x−72+ −r = 0,
x−4 x−4
9
adică f (x) = 0, unde f : R \ {4} → R, f (x) = x3 −18x−72+ −r. Determinăm
x−4
28 Rezolvarea problemelor din numărul anterior

valorile reale ale lui r pentru care ecuat, ia f (x) = 0 are patru solut, ii, utilizând
9
S, irul lui Rolle. Avem f 0 (x) = 3x2 − 18 − . Rădăcinile derivatei sunt
√ (x − 4)2 √
x1 = x2 = 3 s, i x3,4 = 1 ± 2 3. Avem lim f (x) = −∞, f (1 − 2 3) = −r − 44,
x→−∞ √
f (3) = −r − 108, fs (4) = −∞, fd (4) = ∞, f (1 + 2 3) = −r − 44, lim f (x) = ∞,
x→∞
deci S, irul lui Rolle este (−, sgn (−r − 44), sgn (−r − 108), − | +, sgn (−r − 44), +).
Obt, inem că ecuat, ia are 4 solut, ii reale doar ı̂n următoarele două cazuri.
Cazul 1. r = −108, când rădăcinile sunt x1 = x2 = x3 = 3, x4 = −5.
√ √
Cazul 2. r = −44, când rădăcinile sunt x1 = x2 = 1 − 2 3, x3 = x4 = 1 + 2 3.
√ √ √ √
Deci solut, iile sistemului dat sunt (3, 3, 3, −5), (1−2 3, 1−2 3, 1+2 3, 1+2 3)
s, i permutările lor.
Notă. Domnul Alexandru Dan Petrescu din Bucures, ti a propus o rezolvare
asemănătoare ı̂n prima parte, dar bazată apoi pe reprezentarea grafică a funct, iei
x4 − 4x3 − 18x2 + 297
din membrul stâng al ecuat, iei = r, din care s-au observat
x−4
cele două cazuri ı̂n care ecuat, ia are patru solut, ii reale, cazul r = −44 fiind rezolvat
apoi algebric prin substitut, ia x = y + 1, iar cazul r = −108 cu ajutorul primelor
două derivate.
Solut, ia 2 (Istvan Biro, Sânnicolau Mare). Notând a = 4x + 1, b = 4y + 1, c = 4z + 1

 x2+ y +
 z+t=0
s, i d = 4t + 1, sistemul devine x + y + z 2 + t2 = 3
2
, echi-
 x4 + y 4 + z 4 + t4 + x3 + y 3 + z 3 + t3 = 9

 4
 x + y + z = −t
3


xy + xz + yz = t2 −

valent cu 2 . Astfel x, y s, i z sunt rădăcinile ecuat, iei
 3t 3 − 9t + 9
2 4

 xyz =

4t − 3
3t3 − 29 t + 94
 
3 t
X 3 + tX 2 + t2 − X − = 0. Notând X = m − rezultă
2  4t − 3 3
4t2 − 9 7t3 3t2

t 45 9
că m3 + ·m− + + − + = 0. Ecuat, ia va
6 27 4 16 64 64(4t − 3)
avea toate rădăcinile reale dacă s, i numai dacă are ( discriminantul D ≤ 0, adică
2 2 2
2 √ √ )
(2t − 1) (2t + 3) 4t − 3 3 3 1 3
D = 2
≤ 0, deci t ∈ − ,− , , . Conform
576(4t − 3) 2 2 2 2
Formulelor lui Cardano obt, inem următoarele cazuri.
3 1
Cazul 1. t = − . Atunci m1 = m2 = m3 = 0, deci X1 = X2 = X3 = , deci
2 2
a = b = c = 3, d = −5.
√ √ √ √
3 2 3 3 3
Cazul 2. t = − . Atunci m1 = − , m2 = m3 = , deci X1 = m1 + ,
2 3 3 6
Rezolvarea problemelor din numărul anterior 29

3 √ √
X2 = X3 = m2 + , deci a = 1 − 2 3, b = c = 1 + 2 3 s, i permutările lor, iar
√ 6
d = 1 − 2 3.
1 4 2 1
Cazul 3. t = . Atunci m1 = − , m2 = m3 = , deci X1 = m1 − ,
2 3 3 6
1
X2 = X3 = m2 − , deci a = −5, b = c = 3 s, i permutările lor, iar d = 3.
6
√ √ √ √
3 2 3 3 3
Cazul 4. t = . Atunci m1 = , m2 = m3 = − , deci X1 = m1 − ,
2√ 3 3 6
3 √ √
X2 = X3 = m2 − , deci a = 1 + 2 3, b = c = 1 − 2 3 s, i permutările lor, iar
√ 6
d = 1 + 2 3.
(sin x − x cos x) sinn−2 x + xn−2
Z   π
MGO 119. Calculat, i dx, x ∈ 0, , unde
(sin x + x)n 2
n este un număr natural fixat.
Daniel Jinga, Pites, ti
Solut, ie. Integrala din enunt, poate fi scrisă sub forma

(sin x − x cos x) sinn−2 x (sin x − x cos x)xn−2


Z Z
I=  x  n dx +  n dx.
sinn x 1 + sin x
sin x xn 1 +
x

sin x 0
 
 x 0 sin x − x cos x x cos x − sin x
Cum 1 + = 2 s, i 1 + = , se obt, ine
sin x sin x x x2
 0
0 sin x
Z 1+ x

Z 1+
sin x dx − x
I = x n  dx. Rezultă că pentru n = 1 avem
x n
 
1+ sin
sin x 1+
  x
 x  sin x x
I = ln 1 + − ln 1 + + C = ln + C, iar pentru n 6= 1 avem
sin x x sin x
 −n+1
 x −n+1 sin x
1+ 1+
sin x x xn−1 − sinn−1 x
I= − +C = + C.
−n + 1 −n + 1 (n − 1) (x + sin x)n−1
MGO 120. Fie f : [0, 1] → R o funct, ie de două ori derivabilă, cu derivata a doua
continuă, astfel ı̂ncât f (0) = 0, f (1) = (a − 1)2 , f 0 (0) = a2 s, i f 0 (1) = a2 − 4a + 3,
Z 1
2
unde a ∈ R. Demonstrat, i că f 00 (x) dx ≥ 4(4a2 − 6a + 3).
0
Marin Chirciu, Pites, ti
Solut, ie. Conform Inegalităt, ii Cauchy-Buniakowski-Schwarz avem
Z 1 Z 1 Z 1 2
2 00
2 00
(3x − 2a) dx · f (x) dx ≥ (3x − 2a)f (x) dx , (1).
0 0 0
30 Rezolvarea problemelor din numărul anterior
1
(3x − 2a)3 1 (3 − 2a)3 + 8a3
Z
Dar (3x−2a)2 dx = = = 4a2 −6a+3 s, i, integrând
0 9 0 9
Z 1 Z 1
1
prin părt, i, (3x−2a)f (x) dx = (3x−2a)f 0 (x) −
00
3f 0 (x) dx = (3−2a)f 0 (1)+
0 0 0
2af 0 (0) − 3 (f (1) − f (0)) = (3 − 2a)(a2 − 4a + 3) + 2a3 − 3(a − 1)2 = 8a2 − 12a + 6,
Z 1
2
2
deci ı̂nlocuind ı̂n (1) obt, inem că (4a − 6a + 3) f 00 (x) dx ≥ 4(4a2 − 6a + 3)2 .
0
Împărt, ind prin 4a2 − 6a + 3 > 0 rezultă inegalitatea din enunt, .
Remarcăm că egalitatea are loc pentru f (x) = x(x − a)2 .
ARTICOLE S, I NOTE MATEMATICE

În legătură cu Problema 27710 din G.M.-B nr.


6-7-8/2019

Daniel JINGA 1

Problema 27710 din Gazeta Matematică Seria B nr. 6-7-8/2019, propusă la


Clasa a X-a de Rică Zamfir, Bucures, ti, are următorul enunt, :
Să se arate că ı̂n orice triunghi există inegalitatea
1 1 1 1
√ √ 2 + √ √ 2 + √ √ 2 ≤ 4r .
ra + rb rb + rc rc + ra

Ne propunem să generalizăm problema s, i să găsim s, i o inegalitate de sens


contrar. Mai precis demonstrăm următorul rezultat.

Propozit, ia 1. În orice triunghi au loc inegalităt, ile

9 1 1 1 1
≤ √ √ n + √ √ n + √ √ n ≤ n ,
2n (4R + r) n ra + rb
n n rb + rc
n n rc + ra
n 2 ·r

pentru orice n ∈ N, n ≥ 2.

√ √ p√
Demonstraţie. n ra + n rb ≥ 2 ·
ra rb , cu Inegalitatea mediilor.
n

√ √ n √ q
S2 n
Atunci n ra + n rb ≥ 2n · ra rb = 2n · (p−a)(p−b) = √ 2 ·S .
(p−a)(p−b)

Tot cu Inegalitatea mediilor, (p − a)(p − b) ≤ 2p−a−b = 2c .


p
2
√ √ n n+1
Atunci n ra + n rb ≥ 2 c ·S . Se obt, ine √ 1 c
√ n ≤ 2n+1 ·S , de unde prin
( n ra + n rb )
sumare rezultă
X 1 1 p 1
√ √ n ≤ n+1 · 2p = n = n
n r + n r
a b 2 ·S 2 ·p·r 2 ·r

s, i inegalitatea din dreapta este demonstrată.


1
Profesor, Colegiul Nat, ional ,,Ion C. Brătianu”, Pites, ti, jinga.daniel@yahoo.com

31
32 Daniel JINGA

Vom demonstra acum inegalitatea din stânga. Folosind faptul că funct, ia
f : [0, ∞) → R, f (x) = xn , unde n ∈ N, n ≥ 2 este convexă, sau demonstrând prin
induct, ie matematică, avem inegalitatea
x + y n xn + y n
 
≤ , pentru orice x, y ≥ 0 s, i n ∈ N∗ . (1)
2 2
Cu Inegalitatea lui Bergström avem
X 1 9
√ √ n ≥ P √
n r + n r
√ n .
n r + n r
(2)
a b a b

Folosind (1) obt, inem


√ √ n S S
n r + n r
a b ra + rb p−a + p−b S(2p − a − b) S·c
≤ = = = .
2 2 2 2(p − a)(p − b) 2(p − a)(p − b)
√ √ n 2n−1 ·S·c
Prin urmare n ra + n r
b ≤ (p−a)(p−b) .
Rezultă
X √ √ c
( n ra + n rb )n ≤ 2n−1 · S ·
X
. (3)
(p − a)(p − b)
Dar
X c c(p − c) + a(p − a) + b(p − b)
=
(p − a)(p − b) (p − a)(p − b)(p − c)
p(a + b + c) − (a2 + b2 + c2 )
= S2
p
2p2 − 2(p − r2
2 − 4Rr) 2(4R + r) 2(4R + r)
= = = .
r2 p rp S
a2 = 2(p2 − r2 − 4Rr).
P
Am folosit
P √ √ n
Atunci din (3) rezultă n r + n r
a b ≤ 2n (4R + r).
Folosind acum (2) se obt, ine
X 1 9
√ √ n ≥ n
n r + n r
a b 2 (4R + r)
s, i demonstrat, ia este ı̂ncheiată.
Observat, ia 1. Egalităt, ile au loc dacă s, i numai dacă triunghiul este echilateral.
Observat, ia 2. Pentru n = 2 ı̂n inegalitatea din dreapta se obt, ine Problema 27710.

Bibliografie

[1] M. Chirciu, Inegalităt, i cu laturi s, i raze ı̂n triunghi, de la init, iere la performant, ă,
Editura Paralela 45, Pites, ti, 2017.

[2] R. Zamfir, Problema 27710, Gazeta Matematică Seria B, nr. 6-7-8/2019.


O demonstrat, ie inductivă a Inegalităt, ii mediilor

Dorin MĂRGHIDANU 1

În acestă scurtă notă este prezentată o nouă demonstrat, ie, inductivă, a Inega-
lităt, ii mediilor – obt, inută prin utilizarea unei inegalităt, i analitice simple.
Se cunosc numeroase demonstrat, ii pentru inegalitatea clasică dintre media arit-
a1 + a2 + . . . + an √
metică An [a] := s, i media geometrică Gn [a] := n a1 · a2 · . . . · an
n
a numerelor ai ≥ 0, i = 1, n (vezi, de exemplu, [1] – [18]).
În cele ce urmează vom da o nouă demonstrat, ie - prin induct, ie matematică.
Pentru aceasta avem nevoie ı̂n mod esent, ial s, i de următorul rezultat ajutător,
obt, inut prin simple considerat, ii de analiză matematică.
Lema 1. Dacă a > 0, x > 0 s, i m ∈ N∗ , atunci
 m1
am+1

m· + x ≥ (m + 1)a, (1)
x
cu egalitate dacă s, i numai dacă x = a.

 1 m+1 1
am+1 m
Demonstraţie. Fie, f : (0, ∞) → R, f (x) = m · x +x = m·a m · x− m + x.
m+1 m+1
m+1
− m+1 x m −a m
Avem f 0 (x) =1−a m ·x m =m+1 .
x m
Cum f 0 (x) = 0 dacă s, i numai dacă x = a, iar cum evident min f (x) = f (a) =
x>0
(m + 1)a, rezultă inegalitatea s, i condit, ia de egalitate din enunt, .

Observat, ia 1. Inegalitatea din lemă are loc chiar s, i pentru m real, strict pozitiv.

Suntem acum ı̂n măsură să enunt, ăm s, i să demonstrăm următoarea:
Propozit, ia 1 (Inegalitatea mediilor). Pentru orice n ∈ N∗ , xk ≥ 0, k = 1, n,
s, i vectorul x = (x1 , x2 , . . . , xn ) ∈ Rn+ are loc inegalitatea
x1 + x2 + . . . + xn √
An [x] := ≥ n x1 · x2 · . . . · xn =: Gn [x], (2)
n
cu egalitate dacă s, i numai dacă x1 = x2 = . . . = xn .
1
Profesor dr., Colegiul Nat, ional ,,Al. I. Cuza”, Corabia, d.marghidanu@gmail.com

33
34 Dorin MĂRGHIDANU

Demonstraţie. Concluzia este evidentă dacă există k ∈ {1, 2, . . . , n} astfel ı̂ncât


xk = 0. În continuare presupunem că x1 , x2 , . . . , xn > 0.
Vom demonstra inegalitatea prin induct, ie.
Pentru n = 2, relat, ia (2) se obt, ine imediat prin calcul direct, sau din relat, ia
(1) – cu m = 1, x = x2 , a = G2 [x], din care deducem
G22 [x] √
+ x2 ≥ 2 · G2 [x] ⇔ x1 + x2 ≥ 2 · x1 · x2 , (3)
x2

cu egalitate dacă s, i numai dacă x2 = G2 [x] ⇔ x2 = x1 · x2 ⇔ x1 = x2 .
Presupunem inegalitatea (2) adevărată ı̂n cazul a n numere xk > 0, k = 1, n,
cu egalitate dacă s, i numai dacă x1 = x2 = . . . = xn (= Gn [x]) s, i o vom demonstra
s, i ı̂n cazul a n + 1 numere xk > 0, k = 1, n + 1.
Folosind ipoteza de induct, ie vom avea
(n + 1) · An+1 [x] = x1 + x2 + . . . + xn + xn+1 ≥ n · Gn [x] + xn+1 . (4)
Cum !1
Gn+1
n+1 [x]
n

Gn [x] = , (5)
xn+1
atunci pentru membrul drept din (4), folosind Lema 1 cu m = n, x = xn+1 ,
a = Gn+1 [x], avem
!1
Gn+1
n+1 [x] n

n· + xn+1 ≥ (n + 1) · Gn+1 [x]. (6)


xn+1

Din (4), (5) s, i (6) rezultă, finalmente, (n + 1) · An+1 [x] ≥ (n + 1) · Gn+1 [x] ⇔
An+1 [x] ≥ Gn+1 [x].
Egalitatea ı̂n (6) are loc dacă s, i numai dacă
xn+1 = Gn+1 [x] ⇔ xn+1
n+1 = x1 · x2 · . . . · xn · xn+1

⇔ xnn+1 = Gnn [x] ⇔ xn+1 = Gn [x] = x1 = x2 = . . . = xn .

Cu aceasta inegalitatea (2) este complet demonstrată.

Sunt poate mai put, in cunoscute echivalent, ele dintre Inegalitatea mediilor s, i
unele inegalităt, i celebre, precum Inegalitatea lui Young [2], [13], [17], Inegalitatea
lui Bernoulli [2], [11], [13] [17], Inegalitatea lui Hölder [2], [13], [17], Inegalitatea
lui Minkowski [2], [16], Inegalitatea lui Lagrange [7], sau cu alte inegalităt, i mai
put, in celebre [2], [10], [12], [14], [16].
Să precizăm că două inegalităt, i sunt echivalente dacă se implică una pe cealaltă.
În cele ce urmează vom mai evident, ia ı̂ncă o inegalitate echivalentă cu inegali-
tatea mediilor. Mai exact, are loc următoarea:
O demonstrat, ie inductivă a Inegalităt, ii mediilor 35

Propozit, ia 2. Inegalitatea din Lema 1 s, i Inegalitatea mediilor sunt inegalităt, i


echivalente.

Demonstraţie. Implicat, ia – Inegalitatea din Lema 1 ⇒ Inegalitatea mediilor – a


fost prezentată inductiv ı̂n chiar demonstrat, ia propozit, iei anterioare.
Pentru implicat, ia contrară – Inegalitatea mediilor ⇒ Inegalitatea din Lema 1 –
observăm că, pentru m ∈ N∗ , folosind Inegalitatea mediilor avem, succesiv:
 m1  m1  m1
am+1 am+1 am+1
  
m· +x = + ... + +x
x x x
| {z }
m termeni
v" #m
u am+1  m1
u 
m+1
≥ (m + 1) · t ·x
x
r
m+1
m+1 a
= (m + 1) · · x = (m + 1)a,
x
adică tocmai inegalitatea din Lema 1.

Bibliografie

[1] E.F. Beckenbach, R. Bellman, Inequalities, Springer–Verlag, Berlin, 1961.

[2] P.S. Bullen, D.S. Mitrinović, P.M. Vasić, Means and Their Inequalities, D. Reidel
Publidshing Company, Dordrecht/Boston, 1988.

[3] G.H. Hardy, J.E. Littlewood, G. Polya, Inequalities, 2-nd ed., Cambridge University
Press, England, Cambridge (reprinted), 1988.

[4] P.P. Korovkin, Inequalities (engl. translation), Mir Publishers, Moscow, 1975.

[5] D. Mărghidanu, M. Bencze, New Proofs for AM – GM and Pondered AM – GM


Inequalities, Octogon Mathematical Magazine, vol. 12, no. 1, April, 2004, pp. 233-235.

[6] D. Mărghidanu, Trei demonstrat, ii pentru inegalitatea mediilor (Cauchy), SFERA,


III, nr. 5 (1/2004-2005), pp. 1- 4.

[7] D. Mărghidanu, Inegalitatea lui Lagrange este echivalentă cu inegalitatea mediilor,


Arhimede, nr. 3-4, 2005, pp. 17-19.

[8] D. Mărghidanu, M. Bencze, New Means and Refinements for AM-GM-HM Inequalities,
Octogon Mathematical Magazine, vol. 13, no. 2, October, 2005, pp. 999-1001.

[9] D. Mărghidanu, M. Bencze, A new Proof for AM-GM Inequalitiy, Octogon Mathe-
matical Magazine, vol. 13, no. 2, October, 2005, pp. 1021-1026.
36 Dorin MĂRGHIDANU

[10] D. Mărghidanu, O demonstrat, ie a inegalităt, ii mediilor (pornind de la o problema


din Crux Mathematicorum), Revista de Matematica din Timisoara, anul XI (Seria a
IV-a), nr. 1/2006, pp. 6-7.

[11] D. Mărghidanu, O metodă a lui Liouville de demonstrare a inegalităt, ilor, Gazeta


Matematică seria A, Anul XXV (CIV), nr. 1/2007, pp. 17-23.

[12] D. Mărghidanu, Două demonstrat, ii scurte pentru inegalitatea mediilor, Recreat, ii


Matematice seria B, Anul II, nr. 2/2007, pp. 20-21.

[13] D. Mărghidanu, O inegalitate din analiză pentru două demonstrat, ii ale inegalităt, ii
mediilor, revista Minus, nr. 1/2008, pp. 29-30.

[14] D. Mărghidanu, Generalizări ale inegalităt, ilor lui Young, Hölder, Rogers s, i Minkowski,
Gazeta Matematică seria A, Anul XXVI (CV), nr. 3/2008, pp. 208-215.

[15] D. Mărghidanu, D.S. Marinescu, V. Cornea, O inegalitate echivalentă cu inegalitatea


mediilor, Revista de Matematică din Timis, oara, anul III (Seria a IV-a), nr. 3/2008,
pp. 3-6.

[16] D. S. Mitrinović (in cooperation with P. M. Vasiâć), Analytic Inequalities, Band 165,
Springer–Verlag, Berlin–Heidelberg – New-York, 1970.

[17] D. S. Mitrinović, J. E. Pecarić, A.M. Fink, Classical and New Inequalities in Analysis,
Kluwer Acad. Press., 1993.

[18] A. Vernescu, Echivalent, a a s, ase inegalităt, i clasice, Lucrările Seminarului ,,Didactica


matematică”, vol. 14, 1998, pp. 337-346.
Inegalităt, i pentru determinant, ii matricelor de
ordinul 2

1
Florin STĂNESCU

În acest articol sunt prezentate inegalităt, i referitoare la determinant, ii matricelor


din M2 (R). Vor fi folosite atât rezultate care privesc inegalităt, ile algebrice, cât s, i
not, iuni din calculul matriceal, utilizate s, i ı̂n [6].

Aplicat, ia 1. Fie A ∈ M2 (R) o matrice astfel ı̂ncât 2· det A > |tr A| . Demonstrat, i
că oricare ar fi matricea B ∈ M2 (R) pentru care AB = BA, avem inegalitatea
det(A + B) ≥ 0. Când se realizează egalitatea?


 
a11 a12
Solut, ie. Fie A = . Dacă a12 = 0, atunci din |tr A| < 2 · det A ⇒
a21 a22
(a11 + a22 ) <4a11 a22 ⇒(a11 − a22 )2 < 0, contradict, ie. Astfel A =
2
6 λI2 , deci
există numerele reale a, b astfel ı̂ncât B = aA + bI2 . În continuare, det(A +
B) = det [(a + 1) A + bI2 ] = b2 + (a + 1) · tr A · b+(a + 1)2 det2 A. Trinomul de
gradul al doilea in b , b2 + (a+ 1) · tr A · b + (a + 1)2 det2 A, are discriminantul
∆ = (a + 1)2 tr 2 A − 4 · det A ≤ 0, deci det (A + B) ≥ 0. Egalitate avem pentru
a = −1 ⇒ b = 0 ⇒ B = −A.
 
a b
Aplicat, ia 2. Considerăm matricea A ∈ M2 (R), A = . Presupunem că
c d
elementele matricei A satisfac inegalitatea a2 + b2 + c2 + d2 < 51 . Să se arate că
det (A + I2 ) > 0.

Solut, ie. Utilizând inegalitatea ±xy ≥ − 12 x2 + y 2 , putem scrie: det (I2 + A) =




ad+a+d+1−bc ≥ a+d+1− 21 a2 + b2 + c2 + d2 ≥a+d+1− 10 1
≥ 1− 2 5 5 − 10
1

> 0,
1 1
ı̂ntrucât − 5 < a, d < 5 .
√ √

Aplicat, ia 3. Fie A, B ∈ M2 (R) astfel ı̂ncât det (AB + BA) ≤ 0. Să se arate că
det A2 + B 2 ≥ 0.

Solut, ie. Putem scrie: 0 ≤ det (A + B)2 + det (A − B)2

= det A2 + B 2 + AB + BA + det A2 + B 2 − (AB + BA)


  

1
Profesor, S, coala Gimnazială ,,S, erban Cioculescu”, Găes, ti, florin.florinstanescu@yahoo.com

37
38 Florin STĂNESCU

= 2 det A2 + B 2 + det (AB + BA) ,


  

deoarece det (X + Y ) + det (X − Y ) = 2 (det X + det Y ). Cum det (AB + BA) ≤


0, concluzia este imediată.
Aplicat, ia 4. Fie A, B, C ∈ M2 (R) astfel ı̂ncât det A2 + B 2 + C 2 ≤ 0. Să se


demonstreze că

det −A2 + B 2 + C 2 + det A2 − B 2 + C 2 + det A2 + B 2 − C 2 ≥ 0.


  

Solut, ie. Utilizând identitatea

det (X + Y + Z)+det X+det Y +det Z = det (X + Y )+det (Y + Z)+det (Z + X) ,

∀ X, Y, Z ∈ M2 (R), pentru X = −A2 + B 2 + C 2 , Y = A2 − B 2 + C 2 s, i Z =


A2 + B 2 − C 2 obt, inem:

det A2 + B 2 + C 2 + det −A2 + B 2 + C 2 + det A2 − B 2 + C 2 +


  

det A2 + B 2 − C 2 = 4 det2 A + det2 B + det2 C .


 

Cum det A2 + B 2 + C 2 ≤ 0, concluzia se impune.




Aplicat, ia 5. Dacă A, B ∈ M2 (R) s, i det (AB − BA) = 0, atunci au loc următoarele


inegalităt, i:

a) det A2 − B 2 ≤ (det A + det B)2 ;




b) det A2 + B 2 ≥ (det A − det B)2 .




Solut, ie. a) Utilizând identitatea det (A + B) − det A − det B = tr A · tr B − tr (AB),


putem scrie:

(det A + det B)2 − [tr (AB) − tr A · tr B]2


= det2 A + 2 det (AB) + det2 B − [det (A + B) − det A − det B]2
= 2 det (A + B) (det A + det B) − det2 (A + B)
det (A + B) + det (A − B)
= 2 det (A + B) · − det2 (A + B)
2
= det [(A − B) (A + B)] = det A2 − B 2 + AB − BA


= det A2 − B 2 + det (AB − BA) ,




deoarece tr (AB − BA) = tr (A2 − B 2 )(AB − BA) = 0. Prin urmare,


 

det A2 − B 2 = (det A + det B)2 −[tr (AB) − tr A · tr B]2 −det (AB − BA) . (1)


Rezultă că det A2 − B 2 ≤ (det A + det B)2 .




b) Înlocuind ı̂n (1) pe B cu iB obt, inem că det A2 + B 2 = (det A − det B)2 +


[tr (AB) − tr A · tr B]2 + det (AB − BA) ≥ (det A − det B)2 .


Inegalităt, i pentru determinant, ii matricelor de ordinul 2 39

Aplicat, ia 6. Dacă A, B ∈ M2 (R), să se demonstreze ca are loc inegalitatea:

det A2 + B 2 ≥ det (AB − BA) .




Solut, ie. Avem:

0 ≤ |det (A + iB)|2 = det [(A + iB) (A − iB)]

= det A2 + B 2 + i (BA − AB) = det A2 + B 2 − det (AB − BA) ,


  

deoarece tr A2 + B 2 · tr (AB − BA) = 0 = tr A2 + B 2 (AB − BA) . Deci


   

det A2 + B 2 ≥ det (AB − BA) .


Aplicat, ia 7. Fie A, B ∈ M2 (R) astfel ı̂ncât det (AB − BA) ≤ 0. Arătat, i că:
 
1
det (I2 + AB) ≤ det I2 + (AB + BA) .
2

Solut, ie. Considerăm funct, ia

f (x) = det [I2 + AB + x (BA − AB)] = det (I2 + AB) + αx + det (AB − BA) · x2 ,

α, x ∈ R. Avem f (0) = det (I2 +AB) = det (I2 + BA) = f (1).  Astfel, cum f este
concavă, rezultă că f (0) ≤ f 21 , deci det (I2 + AB) ≤ det I2 + 21 (AB + BA) .


Aplicat, ia 8. Se consideră matricele A, B, C ∈ M2 (R) care comută


 două câte
2 2 2
două, iar det (AB + BC + CA) < 0. Arătat, i că det A + B + C ≥ 0.
 2 2 + C 2 + x (AB + BC + CA) =

Solut, ie. Considerăm funct, ia f (x) = det A + B
det A2 + B 2 + C 2 + αx + det (AB + BC + CA) 2

 · x , α, x 2∈ R.  Avem f (−1)  =
det A + B + C − AB − BC − CA = det A + εB + ε C A + ε2 B + εC
2 2 2
2
= det A + εB + ε2 C ≥ 0, unde ε ∈ C \ R, ε3 = 1. Mai departe, avem s, i
f (2) = det (A + B + C)2 ≥ 0. Cum f este concavă, rezultă că s, i f (0) ≥ 0, de unde
obt, inem că det A2 + B 2 + C 2 ≥ 0.


Aplicat, ia 9. Fie A, B ∈ M2 (R) astfel ı̂ncât det (AB + BA) ≤ det (AB − BA) .
Să se arate că det A2 + B 2 ≥ det2 A + det2 B.

Solut, ie. Avem

det A2 + B 2 − det (AB − BA) = det A2 + B 2 + i (BA − AB)


  

= det (A + iB) det (A − iB) = (det A − det B)2 + [tr A · tr B − tr (AB)]2


det(AB − BA) + det(AB + BA)
= det2 A + det2 B − + [tr A · tr B − tr (AB)]2 ,
2
det(AB − BA) − det(AB + BA)
prin urmare det A2 + B 2 − det2 A − det2 B =

+
2
2
[tr A · tr B − tr (AB)] ≥ 0.
40 Florin STĂNESCU
 3
Aplicat, ia 10. Fie A ∈ M2 (R). Să se arate că det A2 + A + I2 ≥ (1−det A)2 .
4

Solut, ie. Considerăm λ1 , λ2 ∈ C valorile proprii ale matricei A. Astfel, putem scrie:
 3 3
det A2 + A + I2 ≥ (1 − det A)2 ⇔ (λ21 + λ1 + 1)(λ22 + λ2 + 1) ≥ (1 − det A)2
4 4
3
⇔ (λ1 λ2 )2 + λ1 λ2 (λ1 + λ2 ) + (λ1 + λ2 )2 − λ1 λ2 + λ1 + λ2 + 1 ≥ (1 − det A)2
4
3 3 3
⇔ det A + det A · tr A + tr A − det A + tr A + 1 ≥ − · det A + · det2 A
2 2
4 2 4
2
⇔ (det A + 2tr A + 1) ≥ 0,

ceea ce este adevărat.

Bibliografie

[1] Gh. Andrei, C. Caragea, Gh. Bordea, Algebră pentru concursurile de admitere s, i
olimpiade s, colare, Editura Topaz, Constant, a, 1993.

[2] M. Andronache, R. Gologan, D. Schwarz, D. S, erbănescu, Olimipiada de matematică


2006-2010, Editura Sigma, Bucures, ti, 2010.

[3] A. Chites, , G. Dospinescu, A. Ismail, G. Kreindler, C. Popa, C. Raicu, A. Zahariuc,


Probleme alese de matematică pentru pregătirea Olimpiadei Nat, ionale, Editura Gil,
Zalău, 2010.

[4] G.H. Golub, C.F. Van Loan, Calculul Matriceal, trad. de A. Cipu s, i M. Cipu, Editura
Theta, Bucures, ti, 2005.

[5] F. Stănescu, Probleme de calcul matriceal. Olimpiade, Concursuri s, colare s, i Bacalau-


reat, Editura Cartea Românească Educat, ional, Pites, ti, 2018.

[6] F. Stănescu, Utilitatea unei formule ı̂n rezolvarea unor probleme de calcul matriceal,
RMGO, nr. 1/2019, pg. 23-26.
O rafinare a Inegalităt, ii lui Turkevich

Leonard Mihai GIUGIUC 1s, i Do Huu Duc THINH 2

În anul 1979, matematicienii rus, i V. Senderov s, i E. Turkevich au publicat ı̂n


revista Kvant următorul rezultat:
4 4
a4i + 2 a2i a2j .
P Q P
Dacă a1 , . . . , a4 ≥ 0, atunci ai ≥
i=1 i=1 1≤i<j≤4

Mai mult, egalitatea are loc dacă s, i numai dacă toate numerele sunt egale sau unul
este nul s, i celelalte trei sunt egale.
Ulterior, acest rezultat a căpătat statutul de Teorema lui Turkevich.
Cât, iva ani mai târziu, matematicianul maghiar Jacob Suranyi publica următorul
rezultat, cunoscut astăzi drept Teorema lui Suranyi :
n n
 n  n 
P n−1
ani +n
P Q P
Dacă a1 , . . . , an ≥ 0, n ≥ 2 atunci (n−1) ai ≥ ai ai .
i=1 i=1 i=1 i=1

Mai mult, egalitatea are loc dacă s, i numai dacă toate numerele sunt egale sau unul
este nul s, i celelalte n − 1 sunt egale.
Ne vom opri la cazul n = 4 al Teoremei lui Suranyi. Inegalitatea lui Suranyi
rafinează inegalitatea lui Turkevich:
 4 4
  4  4  4
P 4 P 3
a2i a2j .
Q P Q P
3 ai + 2 ai ≥ ai ai + 2 ai ≥ 3
i=1 i=1 i=1 i=1 i=1 1≤i<j≤4

În vara anului 2020, Leonard Mihai Giugiuc publică ı̂n revista Recreat, ii Mate-
matice o rafinare a inegalităt, ii lui Suranyi: Dacă a1 , . . . , a4 ≥ 0, atunci
 4 4
 4
 4 2
1
(ai − aj )2
P 4 Q Q P P
3 ai + 2 ai ≥ 18 ai + 3 · ai
i=1 i=1 i=1 i=1 1≤i<j≤4

4 4 4
  
a3i
P P Q
≥ ai +2 ai .
i=1 i=1 i=1

Mai mult, egalitatea are loc dacă s, i numai dacă toate numerele sunt egale sau unul
este nul s, i celelalte trei sunt egale.
1
Profesor, Colegiul Nat, ional ,,Traian”, Drobeta Turnu Severin, leonardgiugiuc@yahoo.com
2
Student, University of Economics Ho Chi Minh City, Vietnam, thinh06032001@gmail.com

41
42 Leonard Mihai GIUGIUC s, i Do Huu Duc THINH

În continuare, vom prezenta o rafinare la dreapta termenului din RHS al


Teoremei lui Suranyi, obt, inând astfel un lant, de 5 inegalităt, i, ce are drept capete
termenii din Teorema lui Turkevich.

Lema 1. Dacă a, b, c ≥ 0 atunci

a2 + b2 + c2 a2 + b2 + c2 + ab + bc + ca ≥ 6 a2 b2 + b2 c2 + c2 a2 .
  

Mai mult, egalitatea are loc pentru (k, k, k), ori (k, k, 0) s, i permutările, k ≥ 0.

Demonstraţie. Rescriem inegalitatea sub forma


2
3 a4 + b4 + c4 + a2 + b2 + c2 a2 + b2 + c2 + ab + bc + ca ≥ 3 a2 + b2 + c2 .
  

Putem presupune WLOG că a ≥ b ≥ c. Conform Cı̂rtoaje EV Theorem, Corollary


1.4 ([2]), pentru funct, ia f (u) = u4 pe (0, ∞), dacă fixăm a + b + c s, i a2 + b2 + c2
(deci implicit ab + bc + ca este fixată), atunci a4 + b4 + c4 este minimal ori pentru
c = 0, ori pentru 0 < c ≤ a = b. Datorită omogenităt, ii, este suficient să considerăm
două cazuri: Cazul 1) c = 0 s, i Cazul 2) a = b = 1 ≥ c > 0.
În Cazul 1) inegalitatea se reduce la a demonstra că a2 + b2 a2 + b2 + ab ≥
 

6a2 b2 , inegalitate evident adevărată din AM-GM: a2 +b2 ≥ 2ab s, i a2 +b2 +ab ≥ 3ab,
cu egalitate dacă s, i numai dacă a = b.
În Cazul 2) inegalitatea se reduce la a demonstra că c(c − 1)2 (c + 4) ≥ 0,
adevărat, cu egalitate dacă s, i numai dacă c = 1, adică a = b = c = 1.

Teorema 1. Dacă a1 , . . . , a4 sunt numere reale nenegative, atunci

4
! 4 
X X X X
a2i 3 a2i + ai aj  ≥ 12 a2i a2j .
i=1 i=1 1≤i<j≤4 1≤i<j≤4

Mai mult, egalitatea are loc dacă s, i numai dacă toate numerele sunt egale sau unul
este nul, iar celelalte trei sunt egale.

Demonstraţie. Rescriem inegalitatea sub forma


! 2
4
 4  4
 4
4
P 2
a2i + a2i
P P P P
6 ai + ai 3 ai aj ≥6 .
i=1 i=1 i=1 1≤i<j≤4 i=1

Presupunem WLOG că a1 ≥ a2 ≥ a3 ≥ a4 .


Facem din nou apel la Cı̂rtoaje EV Theorem, Corollary 1.4 pentru funct, ia
4 4
f (u) = u4 pe (0, ∞), ı̂n ipoteza că fixăm a2i (deci implicit
P P P
ai s, i ai aj
i=1 i=1 1≤i<j≤4
este fixată). Ca s, i ı̂n demonstrat, ia Lemei 1, este de-ajuns să considerăm două
cazuri: Cazul 1) a4 = 0 s, i Cazul 2) a1 = a2 = a3 = 1 ≥ a4 > 0.
O rafinare a Inegalităt, ii lui Turkevich 43

În Cazul 1) inegalitatea se reduce la a demonstra că


 3 !
3

P 2 2 a2i a2j .
P P P
ai 3 ai + ai aj ≥ 12
i=1 i=1 1≤i<j≤3 1≤i<j≤3
!
3 3
 
a2i a2i + a2i a2j . Rămâne de
P P P P
Din Lema 1, 2 ai aj ≥ 12
i=1 i=1 1≤i<j≤3 1≤i<j≤3
! !
3 3
   3  3
a2i a2i +
P P P P 2 P 2 P
arătat că 3 ai aj ≥ 2 ai ai + ai aj ,
i=1 i=1 1≤i<j≤3 i=1 i=1 1≤i<j≤3
3
a2i ≥
P P
adică ai aj , inegalitate evident adevărată, cu egalitate dacă s, i nu-
i=1 1≤i<j≤3
mai dacă a1 = a2 = a3 .
În Cazul 2) inegalitatea se reduce la a demonstra că a4 (a4 − 1)2 (a4 + 3) ≥ 0,
adevărat, cu egalitate dacă s, i numai dacă a4 = 1, adică a1 = . . . = a4 = 1.
Teorema 2. Dacă a1 , . . . , a4 sunt numere reale nenegative, atunci
4
! 4 ! 4 4
! 4 
X X Y 1 X X X
ai a3i + 2 ai ≥ a2i 3 a2i + ai aj  .
4
i=1 i=1 i=1 i=1 i=1 1≤i<j≤4

Mai mult, egalitatea are loc dacă s, i numai dacă toate numerele sunt egale sau unul
este nul, iar celelalte trei sunt egale.

Demonstraţie. Presupunem WLOG că a1 ≥ a2 ≥ a3 ≥ a4 . Utilizând acelas, i


principiu din demonstrat, ia Teoremei 1, considerând funct, iile g(u) = u3 s, i h(u) =
4 4
a3i s, i
P Q
ln u pe (0, ∞), avem că ai sunt minimale ı̂n condit, iile ment, ionate
i=1 i=1
anterior. În Cazul 1) inegalitatea se reduce la a demonstra că
 3  3  3 !
3
 
ai ≥ 14
P 3
a2i a2i +
P P P P
ai 3 ai aj .
i=1 i=1 i=1 i=1 1≤i<j≤3

3
 3
 3
  3
2
a2i a3i a2i
P P P P P
Cum ≥ ai aj , e suficient să arătăm că ai ≥ ,
i=1 1≤i<j≤3 i=1 i=1 i=1
inegalitate adevărată din C.B.S., cu egalitate dacă s, i numai dacă a1 = a2 = a3 .
În Cazul 2) inegalitatea se reduce la a demonstra că a4 (a4 − 1)2 (a4 + 11) ≥ 0,
adevărat, cu egalitate dacă s, i numai dacă a4 = 1, adică a1 = . . . = a4 = 1.
 4  4  4
P P 3 Q
Observat, ia 1. Teoremele 1 s, i 2 ne arată că ı̂ntre termenii ai ai +2 ai
i=1 i=1 i=1
!
 4  4
2 2 1 P 2
a2i +
P P P
s, i 3 ai aj putem insera termenul 4 ai 3 ai aj , ra-
1≤i<j≤4 i=1 i=1 1≤i<j≤4
 4  4  4
P 3
a2i a2j .
P Q P
finând astfel inegalitatea ai ai + 2 ai ≥ 3
i=1 i=1 i=1 1≤i<j≤4
44 Leonard Mihai GIUGIUC s, i Do Huu Duc THINH

Cu aceasta, demonstrarea lant, ului de 5 inegalităt, i având drept capete termenii


din Teorema lui Turkevich este completă.

În ı̂ncheiere, propunem cititorilor spre studiu următoarele probleme, pentru


a1 , . . . , a4 ≥ 0:
1) Considerăm următoarele rafinări ale Inegalităt, ii lui Turkevich:
i) (Vietnam, 1997)
!2β−2
4 4 4 4 4
a4i + 2 a4i + 2 ai β ≥ a2i a2j ,
P Q P Q P
ai ≥ 4
·
i=1 i=1 i=1 i=1 1≤i<j≤4
a2i
P
i=1

unde 1 < β ≤ 2;
ii) (L.M. Giugiuc s, i D.H.D. Thinh, 2020)
!
4 4
 4  4
1
a4i + 2
P 2
a2i + a2i a2j .
P Q P P P
ai ≥ 12 ai 3 ai aj ≥
i=1 i=1 i=1 i=1 1≤i<j≤4 1≤i<j≤4

Există o relat, ie generală de ordine ı̂ntre termenii mediani ai celor două rafinări?
În caz afirmativ, determinat, i β.
2) Este adevărată inegalitatea
 √3

3 a4 + b4 + c4 + 3 a4 b4 c4 ≥ a2 + b2 + c2 a2 + b2 + c2 + ab + bc + ca
 

pentru orice numere reale nenegative a, b s, i c?


3) Considerăm următoarele lant, uri de inegalităt, i:
4 P 2 2
  4  4
 4  4
P 3
a2i a2j ;
P Q Q P
ai ai + 2 ai ≥ ai +2 ai ≥ 3
i=1
4
 i=1
4
 i=1
4
i=1
4
  4i=1 1≤i<j≤4

3 1 2 2 a2i a2j .
P P Q P P P P
ai ai +2 ai ≥ 4 ai 3 ai + ai aj ≥ 3
i=1 i=1 i=1 i=1 i=1 1≤i<j≤4 1≤i<j≤4

Există o relat, ie generală de ordine ı̂ntre termenii mediani ai celor două lant, uri?

Bibliografie

[1] https://www.ams.org/journals/proc/2012-140-03/S0002-9939-2011-10986-9/
S0002-9939-2011-10986-9.pdf

[2] https://www.emis.de/journals/JIPAM/images/059_06_JIPAM/059_06_www.pdf?fbclid=
IwAR0vNeJQIMPHu1l0prIUVXiYmdCS3Dh0tcmBnKiBUCFMeLmbpOo8hEz6W_s
TESTE PENTRU EXAMENE

Teste pentru examenul de Evaluare Nat, ională

Costel ANGHEL 1 s, i Florea BADEA 2

Testul 1

SUBIECTUL I
Încercuit, i litera corespunzătoare răspunsului corect.

1
1. Rezultatul calculului 5 · 3 · + 8 · 8−1 · 4 este: (5p)
3
a) 6; b) 7; c) 8; d) 9.

2. Diferent, a dintre cel mai mare s, i cel mai mic număr ı̂ntreg din intervalul
(−6; 10] este egală cu: (5p)

a) 4; b) 15; c) 1,6; d) 9.

2x + 3
3. Dacă = 2, atunci x este egal cu: (5p)
4
a) 2,5; b) 3; c) 2,05; d) 2,4.

4. Numărul natural de forma ab care verifică relat, ia ab1 + 2ab + ab0 = 936 este
egal cu: (5p)

a) 34; b) 36; c) 35; d) 33.


√ √
12 5 − 12 7
5. Numărul √ √ este: (5p)
7− 5

1
Profesor, Colegiul Nat, ional ,,Ion Minulescu”, Slatina, anghelcostel2012@yahoo.com
2
Profesor, S, coala Gimnazială ,,Nicolae Coculescu”, Scornices, ti

45
46 Costel ANGHEL s, i Florea BADEA

a) ı̂ntreg negativ; c) rat, ional pozitiv;


b) natural; d) irat, ional.

√ √ √ √
6. Se consideră mult, imea M = { 4; 14; 24; . . . ; 404}. Numărul de ele-
mente din M care sunt numere naturale este egal cu: (5p)

a) 6; b) 7; c) 8; d) 4.

SUBIECTUL al II-lea
Încercuit, i litera corespunzătoare răspunsului corect.

1. Pe dreapta d se consideră punctele A, B, C, D, ı̂n această ordine, BC = 3AB


s, i C este mijlocul lui [AD]. Cât la sută reprezintă AB din AD? (5p)

a) 10%; b) 12%; c) 12,5%; d) 11,5%.

2. În triunghiul ABC, ^A = 90◦ , AB = 6 cm s, i AC = 8 cm. Mediana


corespunzătoare ipotenuzei are lungimea: (5p)

a) 4 cm; b) 5 cm; c) 4,5 cm; d) 6 cm.

3. Dacă ABCD este paralelogram, AB = 5 cm s, i BC = 6,52 cm, atunci


perimetrul lui ABCD este egal cu: (5p)

a) 22,14 cm; b) 23 cm; c) 21,14 cm; d) 23,04 cm.

4. În triunghiul ABC, ^BAC = 110◦ , ^ACB = 25◦ s, i AD = x, AD ⊥ BC,


D ∈ BC. Lungimea lui [AB] este: (5p)
x √ √
a) 2x; b) ; c) x 2; x 3
2 d) .
2

5. Un paralelipiped dreptunghic are dimensiunile 8,5 cm, 6,5 cm s, i 3,5 cm.


Suma dimensiunilor acestui paralelipiped dreptunghic este egală cu: (5p)

a) 74 cm; b) 64,5 cm; c) 18,15 cm; d) 18,5 cm.

6. O prismă patrulateră regulată are latura bazei de 5 cm s, i volumul de 100


cm3 . Înălt, imea prismei are lungimea: (5p)

a) 3,5 cm; b) 4 cm; c) 4,5 cm; d) 5 cm.


Teste pentru examenul de Evaluare Nat, ională 47

SUBIECTUL al III-lea
Scriet, i pe foaia de examen rezolvările complete.

1. Se consideră mult, imea A = {a ∈ N|a = 10n + 7, n ∈ N}.

a) Câte elemente din A sunt mai mici ca 1000? (2p)


b) Determinat, i A ∩ Q. Justificat, i răspunsul. (3p)
 
3 + x 2 + 3x 6x
2. Fie expresia E(x) = − · .
4x 6x 10 − 6x

a) Determinat, i mult, imea D, domeniul de existent, ă al expresiei E(x). (2p)


b) Arătat, i că E(x) este constantă pe D. (3p)

3. Fie ABCD dreptunghi, AC ∩ BD = {O}, AC = 40 cm s, i m(^COD) = 120◦ .


Aflat, i:

a) Perimetrul dreptunghiului ABCD. (2p)


b) Aria patrulaterului determinat de mijloacele segmentelor (OA), (OB),
(OC) s, i (OD). (3p)

4. Fie ABCD un dreptunghi, punctele E, F ∈ [CD], AB = 24 cm, BC = 18


AB CD
cm, CE = s, i DF = . Dreapta AE intersectează pe BF ı̂n P , iar
3 4
AP ∩ BC = {M } s, i BP ∩ AD = {N }.

a) Calculat, i lungimile segmentelor [DN ] s, i [CM ]. (2p)


b) Calculat, i aria 4P EF . (3p)

5. În rombul ABCD, AB = 18 cm, m(^BAD) = 60◦ s, i AC ∩ BD = {O}.


Dreptele AE s, i BF sunt perpendiculare pe planul rombului s, i AE = BF =
BD.

a) Arătat, i că BD ⊥ (EOC). (2p)


b) Calculat, i distant, a de la punctul O la dreapta EF . Studiat, i toate
cazurile posibile. (3p)

6. Fie ABCDA0 B 0 C 0 D0 o prismă patrulateră regulată cu AA0 = 8 2 cm.
Sect, iunea axială care cont, ine muchia [CC 0 ] are aria egală cu 128 cm2 .

a) Aflat, i lungimea muchiei bazei. (2p)


b) Demonstrat, i că oricare ar fi punctul P pe muchia (BB 0 ), distant, a de la
P la planul sect, iunii axiale este aceeas, i (constantă) s, i calculat, i-o. (3p)
48 Costel ANGHEL s, i Florea BADEA

Testul 2

SUBIECTUL I
Încercuit, i litera corespunzătoare răspunsului corect.

1. Cel mai mare număr natural prim mai mic ca 100 este: (5p)

a) 91; b) 97; c) 95; d) 99.

2. Sfertul numărului 134 este numărul: (5p)

a) 32,5; b) 34; c) 33,5; d) 135.

a+2 a+3
3. Dacă = , atunci a este egal cu: (5p)
5 10
a) 1; b) -2; c) 3; d) -1.

4. Extrăgând rădăcina pătrată din fiecare număr natural de trei cifre, vom
obt, ine ca rezultat un număr irat, ional de un număr de ori egal cu: (5p)

a) 532; b) 878; c) 694; d) 880.


√ √
5. Dintre numerele a = 2 + 3, b = 2 − 3, c = 2 s, i d = 1, cel care reprezintă
media geometrică a două dintre celelalte trei este: (5p)

a) c; b) d; c) b; d) a.
√ √ √
6. Numărul real x care verifică relat, ia x 32 − 18 = 162 este egal cu: (5p)

a) 3; b) 2; c) 4; d) 5.

SUBIECTUL al II-lea
Încercuit, i litera corespunzătoare răspunsului corect.

1. În 4ABC, BC = AB s, i AB = 2 · AC. Dacă BC = 10 cm, atunci perimetrul


triunghiului ABC este: (5p)

a) 24 cm; b) 25 cm; c) 24,05 cm; d) 8 cm.

2. Fie ABCD un paralelogram, AC ∩ BD = {O} s, i M mijlocul lui [AB]. Dacă


AB = 6 cm s, i OM = 4 cm, atunci perimetrul lui ABCD este: (5p)
Teste pentru examenul de Evaluare Nat, ională 49

a) 27 cm; b) 29 cm; c) 28 cm; d) 30 cm.



3. Un 4ABC echilateral are aria 4 3 cm2 . Latura sa are lungimea: (5p)

a) 4 cm; b) 4,02 cm; c) 5 cm; d) 3,5 cm.

4. Un trapez isoscel s, i ortodiagonal are aria 144 cm2 . Lungimea liniei mijlocii
a trapezului este egală cu: (5p)

a) 10 cm; b) 11 cm; c) 13 cm; d) 12 cm.



5. Un cub are diagonala 6 6 cm. Suma tuturor muchiilor cubului este egală
cu: (5p)
√ √ √
a) 71 2 cm; b) 70 3 cm; c) 72 2 cm; d) 74 cm.

6. Un paralelipiped dreptunghic are diagonala d = 5 30 cm s, i două dintre
dimensiuni egale cu 10 cm s, i respectiv 5 cm. Cealaltă dimensiune este egală
cu: (5p)

a) 24 cm; b) 25 cm; c) 32 cm; d) 50 cm.

SUBIECTUL al III-lea
Scriet, i pe foaia de examen rezolvările complete.

1. Două numere naturale a s, i b sunt astfel ı̂ncât 70% din a eate egal cu 30%
din b.

a) Arătat, i că a s, i b sunt direct proport, ionale cu 3 s, i 7. (2p)


b) S, tiind că diferent, a pătratelor celor două numere este egală cu 1000,
determinat, i numerele a s, i b. (3p)

2. Se dă funct, ia f : R → R, f (x) = 5ax + 2a − 3, unde a este număr real.


1
a) Pentru a = , reprezentat, i grafic funct, ia f . (2p)
2  
2
− ; −3 apart, ine
b) Demonstrat, i că pentru orice număr real a punctul P
5
graficului funct, iei f . (3p)
√ √ q √ 2 q √ 2
3. Se dau numerele a = |4− 10|+2( 10−4) s, i b = 6−4 + 2 6−3 .

a) Arătat, i că a s, i b sunt irat, ionale. (2p)


b) Stabilit, i dacă (b − a − 5)(a + b + 3) este natural. Justificat, i. (3p)
50 Costel ANGHEL s, i Florea BADEA

4. În paralelogramul ABCD, AB = BD = 20 cm s, i m(^CBD) = 60◦ .

a) Calculat, i lungimea diagonalei [AC]. (2p)


b) Aflat, i distant, a de la B la CD. (3p)

5. Fie ^AOB, m(^AOB) = 60◦ s, i OA = OB = 10 cm. În punctul C al


bisectoarei unghiului ^AOB, se ridică perpendiculara CD pe planul (OAB).
Dacă OC = 18 cm, se cer:

a) Arătat, i că triunghiurile DOA s, i DOB sunt echivalente. (2p)


b) Aflat, i sinusul unghiului plan corespunzător diedrului dintre planele
(DAC) s, i (DBC). (3p)

6. Îm piramida regulată V ABCD, punctul O este centrul bazei iar M mijlocul
lui (BC). S, tiind că AD = 16 cm s, i m(^V M O) = 60◦ , se cer:

a) Volumul piramidei. (2p)


b) Distant, a de la punctul C la planul (V AD). (3p)

Testul 3

SUBIECTUL I
Încercuit, i litera corespunzătoare răspunsului corect.

1. Cel mai mic pătrat perfect de patru cifre este: (5p)

a) 1000; b) 1021; c) 1025; d) 1024.

2. Suma tuturor numerelor de forma a5b care verifică egalitatea a + b = 4 este


egală cu: (5p)

a) 1106; b) 1306; c) 1206; d) 1208.

a + 10
3. Fie numărul a ∈ N astfel ı̂ncât ∈ N. Numărul a este egal cu: (5p)
a+3
a) 2; b) 4; c) 7; d) 1.
√ √
4. Dacă y ∈ Z s, i 26 < y < 43 atunci y poate fi: (5p)

a) 5; b) 6; c) 13; d) 31.

1 9
5. Fie funct, ia f : R → R, f (x) = 3x − . Valoarea funct, iei f ı̂n punctul x =
2 6
este egală cu: (5p)
Teste pentru examenul de Evaluare Nat, ională 51

a) 4; b) 2; 5 8
c) ; d) .
6 3

6. Mult, imea solut, iilor inecuat, iei 33x − 363 ≤ 0 cont, ine n numere naturale.
Atunci n este egal cu: (5p)

a) 10; b) 11; c) 12; d) 25.

SUBIECTUL al II-lea
Încercuit, i litera corespunzătoare răspunsului corect.

1. Perimetrul unui pătrat cu latura de 1,5 m este: (5p)

a) 3 m; b) 4,5 m; c) 6 m; d) 8 m.

2. Un romb are lungimile diagonalelor de 2,5 m s, i 4 m. Aria acestui romb este


egală cu: (5p)

a) 10 m2 ; b) 5 m2 ; c) 6,5 m2 ; d) 4,5 m2 .


3. Diagonala unui cub are lungimea 10 3 dm. Volumul cubului, ı̂n dm3 , este
egal cu: (5p)

a) 100; b) 900; c) 300; d) 1000.

4. O linie mijlocie a unui triunghi echilateral are lungimea 12 cm. Aria acestui
triunghi, ı̂n cm2 , este: (5p)
√ √ √
a) 144 3; b) 124 3; c) 24 2; d) 144.

5 3
5. Un tetraedru regulat are apotema unei fet, e egală cu cm. Suma tuturor
2
muchiilor tetraedrului dat este egală cu: (5p)

a) 92 cm; b) 9 dm; c) 90 mm; d) 80 cm.

6. Paralelogramul ABCD, ı̂n care m(^ADB) = 65◦ , este romb dacă s, i numai
dacă m(^ACB) este: (5p)

a) 15◦ ; b) 30◦ ; c) 35◦ ; d) 25◦ .


52 Costel ANGHEL s, i Florea BADEA

SUBIECTUL al III-lea
Scriet, i pe foaia de examen rezolvările complete.

1. Într-o urnă sunt bile albe s, i ros, ii, ı̂n total 75 de bile. Probabilitatea să se
4
extragă o bilă albă din urnă este de .
5
a) Aflat, i numărul de bile albe din urnă. (2p)
b) Calculat, i probabilitatea ca din urnă să se extragă o bilă ros, ie. (3p)
q √ 2 q √ 2
2. Numerele reale x s, i y verifică relat, ia x−5 7 + 7 5 − y ≤ 0.

a) Aflat, i numerele x s, i y. (2p)


2 1
b) Comparat, i numerele s, i √ . (3p)
3x + 5y 15xy
3. a) Rezolvat, i ecuat, ia x2 − 16x + 64 = 0, x ∈ R. (2p)
b) Determinat, i numerele reale x s, i y, s, tiind că ele verifică egalitatea
2x2 + 4y 2 + 64 = 16x + 4xy. (3p)

4. Fie 4ABC isoscel, AB = AC = 100 cm s, i BC = 120 cm. Cercul ı̂n care


[BC] este diametru intersectează (AB) ı̂n M s, i (AC) ı̂n N .

a) Dacă P este mijlocul lui (BC), demonstrat, i că dreptele AP , BN s, i


CM sunt concurente. (2p)
b) Calculat, i aria patrulaterului BCN M . (3p)

5. Piramida patrulateră regulată √ V ABCD are toate muchiile congruente,


ı̂nălt, imea [V O] are lungimea 10 2 cm s, i punctul M este mijlocul muchiei
(V A). Calculat, i:

a) Volumul piramidei V ABCD. (2p)


b) Distant, ele de la punctul M la diagonalele bazei piramidei. (3p)

6. Se consideră un cilindru circular drept, ı̂n care ABCD este o sect, iune axială.
Generatoarea cilindrului are lungimea de 25 cm. În baza care cont, ine
diametrul
√ [AB] se ı̂nscrie un triunghi echilateral AM N a cărui arie este egală
cu 100 3 cm2 . Aflat, i:

a) Raza s, i volumul cilindrului. (2p)


b) Distant, a de la punctul D la dreapta M N . (3p)
Teste pentru examenul de Bacalaureat,
specializarea s, tiint, e ale naturii

Mihai Florea DUMITRESCU 1

Testul 1

SUBIECTUL I (30 de puncte)

1. Se consideră numărul complex z = a + i. Aflat, i a ∈ R astfel ı̂ncât z · z̄ = a + 3.

2. Determinat, i cea mai mare valoare ı̂ntreagă a funct, iei f : R → R, f (x) =


−x2 + x + 6.

3 1
3. Rezolvat, i ı̂n mult, imea numerelor reale ecuat, ia 2x = √ .
2
√
4
4. Calculat, i probabilitatea ca, alegând aleator un număr din mult, imea 2,
√4
√ √
3, 4 4, . . . , 4 2021 , acesta să fie rat, ional.

5. Determinat, i a ∈ R pentru care vectorii ~u = (a + 1)~i + 2~j s, i ~v = 6~i − 5~j sunt


necoliniari.

6. Fie triunghiul ABC cu AB = 6, AC = 9 s, i BC = 3 7. Calculat, i
arccos (cos A).

SUBIECTUL al II-lea (30 de puncte)

 
5 2
1. Se consideră matricea A = .
2 1

a) Calculat, i det A.
b) Aflat, i inversa matricei 6I2 − A.
c) Rezolvat, i ı̂n M2 (R) ecuat, ia X 2 = A.
1
Profesor, Liceul ,,S, tefan Diaconescu”, Potcoava, florin14mihai@yahoo.com

53
54 Mihai Florea DUMITRESCU

2. Pe mult, imea numerelor reale se defines, te legea de compozit, ie asociativă

1
x ∗ y = xy − (x + y) + 14.
7
1
a) Arătat, i că x ∗ y = (x − 7)(y − 7) + 7, pentru orice numere reale x s, i y.
7
b) Arătat, i că intervalul (7, +∞) este parte stabilă a lui R ı̂n raport cu
legea de compozit, ie ,,∗”.
c) Rezolvat, i ı̂n mult, imea numerelor reale ecuat, ia x∗(−x)∗x = (x − 7)2 +7.

SUBIECTUL al III-lea (30 de puncte)

1
1. Se consideră funct, ia f : (−∞, 2) → R, f (x) = .
ex (x − 2)
a) Calculat, i lim f (x).
x→−∞
b) Determinat, i ecuat, ia asimptotei verticale la graficul funct, iei f .
1
c) Arătat, i că x ≥ 2 − x−1 , pentru orice x ∈ (−∞, 2).
e

2. Fie funct, iile f : (1, +∞) → R, f (x) = ln ( x − 1) s, i F : (1, +∞) → R,
√ x √
F (x) = (x − 1) ln ( x − 1) − − x.
2
a) Arătat, i că funct, ia F este o primitivă a funct, iei f .
Z e+1
f x2 dx = 1.

b) Arătat, i că
2
Z e2
f (x) + xf 0 (x) dx.

c) Calculat, i
4

Testul 2

SUBIECTUL I (30 de puncte)


1. Aflat, i partea ı̂ntreagă a numărului a = 1 + 2 3.

2. Determinat, i numărul ı̂ntreg a, pentru care minimul funct, iei f : R → R,


2
 2
f (x) = a + 1 x + 6x + 1 este număr ı̂ntreg.

3. Rezolvat, i ı̂n mult, imea numerelor ı̂ntregi inecuat, ia log2 (x2 − 3x + 2) ≤ 1.

4. Calculat, i probabilitatea ca, alegând un număr din mult, imea numerelor


naturale de două cifre, acesta să aibă produsul cifrelor un număr par.
Teste pentru examenul de Bacalaureat, specializarea s, tiint, e ale naturii 55

5. În reperul cartezian xOy se consideră punctele A(0, 3) s, i B(a, 0), a ∈ R∗ .


Calculat, i a astfel ı̂ncât panta dreptei AB să fie egală cu 2.
1
6. Rezolvat, i ı̂n intervalul (0, π) ecuat, ia cos2 x = .
2

SUBIECTUL al II-lea (30 de puncte)


x+y+z =9

1. Se consideră sistemul de ecuat, ii 2x − ay + z = −2 , unde a ∈ R.
ax + 5y − 2z = 8

a) Calculat, i determinantul matricei sistemului.


b) Arătat, i că matricea sistemului este nesingulară, pentru orice a ∈ R.
c) Pentru a = 3, rezolvat, i sistemul.

2. Se consideră polinomul f = (X + 1)n + X − 9, n ∈ N∗ .

a) Pentru n = 4, aflat, i restul ı̂mpărt, irii polinomul f la polinomul X + 2.


b) Pentru n = 3, aflat, i rădăcinile polinomului f .
c) Pentru n = 3, aflat, i un polinom g de gradul al III-lea cu rădăcinile
1 1 1
y1 = − , y2 = − , y3 = − , unde x1 , x2 , x3 sunt rădăcinile
x1 x2 x3
polinomului f .

SUBIECTUL al III-lea (30 de puncte)


1. Se consideră funct, ia f : (0, +∞) → R, f (x) = x x − ln x.
3√ 1
a) Arătat, i că f 0 (x) = x − , pentru orice x ∈ (0, +∞).
2 x
b) Determinat, i ecuat, ia tangentei la graficul funct, iei f ı̂n punctul de abscisă
x0 = 1 situat pe graficul funct, iei f .
 
2 2
c) Demonstrat, i că f (x) ≥ 1 − ln , pentru orice x ∈ (0, ∞).
3 3

2. Se consideră funct, ia f : R → R, f (x) = x2 + 2x + 4.


Z 1
4
a) Arătat, i că (f (x) − 4) dx = .
0 3
Z 1
|x| ex f (x) − x2 − 2x dx.

b) Calculat, i
−1
Z 1
2 1 2
c) Arătat, i că ≤ dx ≤ .
7 −1 f (x) 3
56 Mihai Florea DUMITRESCU

Testul 3

SUBIECTUL I (30 de puncte)

1. Aflat, i produsul primilor 5 termeni ai unei progresii geometrice, s, tiind că


termenul al treilea este egal cu 2.

2. Determinat, i numărul natural a, s, tiind că parabola y = (a + 1) x2 + 5x + a2


cont, ine punctul A (1, 8).

3. Rezolvat, i ı̂n mult, imea numerelor reale ecuat, ia |2x − 1| − |5 − 2x | = 0.

4. Determinat, i probabilitatea ca, alegând o submult, ime cu două elemente din


mult, imea {1, 2, 3, . . . , 7}, aceasta să aibă ambele elemente cifre pare.

5. În reperul cartezian xOy se consideră punctul A(5, 2) s, i dreapta h de ecuat, ie


x − y + 2 = 0. Scriet, i ecuat, ia perpedicularei din punctul A pe dreapta h.

6. Un triunghi ABC are laturile AB = 4, AC = 6 s, i BC = 9. Calculat, i


produsul dintre raza cercului ı̂nscris s, i raza cercului circumscris triunghiului
ABC.

SUBIECTUL al II-lea (30 de puncte)

 
a a+1
1. Fie matricea A (a) = , a ∈ R.
a−1 a

a) Arătat, i că det (A (a))2 = 1, pentru orice număr real a.


b) Arătat, i că (A (a))3 = 4a2 − 1 A (a) − 2aI2 , pentru orice număr real a.


c) Rezolvat, i ı̂n M2 (R) ecuat, ia A (1) · X = A (−2).

2. Pe mult, imea numerelor reale se defines, te legea de compozit, ie

x ∗ y = xy − 3x − 3y + 12.

a) Aflat, i elementul neutru al legii ,,∗”.


b) Aflat, i numerele ı̂ntregi ale căror simetrice ı̂n raport cu legea ,,∗” sunt
tot numere ı̂ntregi.
c) Arătat, i că f (xy) = f (x) ∗ f (y), oricare ar fi numerele reale x s, i y, unde
f : R → R, f (x) = x + 3.

SUBIECTUL al III-lea (30 de puncte)

√ √
1. Se consideră funct, ia f : (1, +∞) → R, f (x) = ln ( x − 1) − x.
Teste pentru examenul de Bacalaureat, specializarea s, tiint, e ale naturii 57

f (x) − f (2)
a) Calculat, i lim .
x→2 x2 − 4
b) Determinat, i asimptota verticală la graficul funct, iei f .
c) Arătat, i că f (x) + 2 ≤ 0, pentru orice x ∈ (1, +∞).
 
1
2. Se consideră funct, ia f : (0, +∞) → R, f (x) = ln x + .
x
Z 2
a) Calculat, i f 0 (x) dx.
1
Z 2
f (x) − ln x2 + 1

b) Calculat, i dx.
1
Z 2
c) Calculat, i f (x) dx.
1
Teste pentru examenul de Bacalaureat,
specializarea matematică-informatică

Costel BĂLCĂU 1

Testul 1

SUBIECTUL I (30 de puncte)


√ √ 2 √ √ 2
1. Arătat, i că numărul a = 3+i 5 + 3 − i 5 este ı̂ntreg.

2. Determinat, i cel mai mic număr ı̂ntreg k pentru care solut, iile ecuat, iei
2x2 − 7x − k = 0 sunt numere reale.

3. Rezolvat, i ı̂n mult, imea numerelor reale ecuat, ia log3 (9x) + logx 3 = 4.

4. Câte elemente are o mult, ime care are exact 67 de submult, imi cu cel mult
două elemente?

5. În reperul cartezian xOy se consideră punctele A(−1, −1), B(1, 2) s, i C(2, 0).
Calculat, i aria triunghiului ABC.
h πi π  π  √
6. Aflat, i x ∈ 0, pentru care 2 cos x sin − x + 2 sin x cos + x = 2.
2 2 2

SUBIECTUL al II-lea (30 de puncte)


 
1 0 0
1. Pentru orice x ∈ R se consideră matricea A(x) =  2x 1 0 .
2
3x + 5x 3x 1

a) Calculat, i det (A(2020)).


b) Arătat, i că A(x)A(y) = A(x + y), pentru orice x, y ∈ R.
c) Determinat, i matricea X ∈ M3 (R) pentru care A(7) · X · A(8) = A(25).

2. Pe mult, imea numerelor reale se defines, te legea de compozit, ie asociativă

x ∗ y = 3xy − 2x − 2y + 2.
1
Conf. univ. dr., Universitatea din Pites, ti, cbalcau@yahoo.com

58
Teste pentru examenul de Bacalaureat, specializarea matematică-informatică 59
  
2 2 2
a) Arătat, i că x ∗ y = 3 x − y− + , pentru orice x, y ∈ R.
3 3 3
b) Rezolvat, i ûn mult, imea numerelor reale ecuat, ia x ∗ (−x) = −10.
c) Determinat, i numărul natural n pentru care are loc egalitatea
     
2 2 2 2
3n + ∗ 3n+1 + ∗ . . . ∗ 3n+9 + = 8121 + .
3 3 3 3

SUBIECTUL al III-lea (30 de puncte)

1 1
1. Se consideră funct, ia f : R → R, f (x) = − .
x2 − 2x + 2 x2 + 1
 
1
a) Calculat, i f0 .
2
b) Determinat, i ecuat, ia dreptei care trece prin punctul A(2, −1) s, i este
perpendiculară pe tangenta la graficul funct, iei f duse prin punctul de
abscisă x0 = 1 situat pe graficul funct, iei f .
3
c) Calculat, i lim (f (1) + f (2) + . . . + f (n))n .
n→∞

2. Se consideră funct, ia f : [−2, 2] → R, f (x) = 4 − x2 .
Z 0
x
a) Calculat, i √ dx.
− 3 f (x)
Z √3
b) Calculat, i f (x) dx.
0
c) Demonstrat Z, ixcă există un unic număr x ∈ [−2, 2] pentru care are loc
egalitatea 2020−f (t) dt = x.
0

Testul 2

SUBIECTUL I (30 de puncte)


√ √ √ √
1. Calculat, i a = log6 ( 2 + 3) − log6 (3 2 + 2 3).

2. Determinat, i punctele de intersect, ie dintre graficele funct, iilor f, g : R → R,


f (x) = 22x+1 + 4 s, i g(x) = 9 · 2x .

x2 x
3. Rezolvat, i ı̂n mult, imea numerelor reale ecuat, ia 2
= .
x −1 2x − 2
4. Calculat, i probabilitatea ca, alegând un număr din mult, imea numerelor
naturale de două cifre, acesta să aibă produsul cifrelor un număr par.
60 Costel BĂLCĂU

5. În reperul cartezian xOy se consideră punctele A(1, −2), B(4, 2), C(0, 3) s, i
−→ −−→
D astfel ı̂ncât ABCD este paralelogram. Calculat, i AC − BD .
 π
6. Determinat, i x ∈ [0, 2π), s, tiind că 5 sin (x − π) + 3 cos x − = 1.
2

SUBIECTUL al II-lea (30 de puncte)


 
a 1 1
1. Fie matricea A(a, b) =  2 b 2 , unde a, b ∈ R.
1 1 a
a) Determinat, i a ∈ R pentru care matricea A(a, 1) nu este inversabilă.
b) Determinat, i m ∈ R s, tiind că matricea A(m, 2m) are rangul egal cu 2.
c) Determinat, i perechile de numere ı̂ntregi (a, b) astfel ı̂ncât matricea
adjunctă a matricei A(a, b) să aibă determinantul egal cu 1.

2. Se consideră polinomul f = X 4 − 3X 3 + X 2 + m, unde m ∈ R.

a) S, tiind că x1 , x2 , x3 s, i x4 sunt rădăcinile polinomului f , determinat, i


x3 + 1 x32 + 1 x33 + 1 x34 + 1
m ∈ R∗ pentru care 1 + + + = x1 x2 x3 x4 .
x1 x2 x3 x4
b) Determinat, i m ∈ Z astfel ı̂ncât polinomul f să aibă o rădăcină dublă.
c) Determinat, i m ∈ R pentru care toate rădăcinile polinomului f sunt
numere reale.

SUBIECTUL al III-lea (30 de puncte)

2
1. Se consideră funct, ia f : R → R, f (x) = 3x + .
ex
a) Determinat, i asimptotele graficului funct, iei f .
b) Determinat, i coordonatele punctului situat pe graficul funct, iei f ı̂n care
tangenta la graficul funct, iei f este paralelă cu prima bisectoare.
c) Determinat, i a ∈ R, s, tiind că f (x) ≥ ax + 2 pentru orice x ∈ R.

2. Se consideră funct, ia f : (0, ∞) → R, f (x)Z= ln x s, i, pentru orice n ∈ N∗ s, i


a
f (x)
a ∈ (0, ∞), se consideră numărul In (a) = dx.
1 xn
a) Calculat, i 4I1 (e) + eI2 (e).
b) Calculat, i aria suprafat, ei plane delimitate de graficul funct, iei f , axa Ox
1
s, i dreptele de ecuat, ii x = s, i x = e.
e
I2020 (a)
c) Calculat, i lim .
a→1 (a − 1)2
Teste pentru examenul de Bacalaureat, specializarea matematică-informatică 61

Testul 3

SUBIECTUL I (30 de puncte)

√ √
1. Calculat, i a = |3 + 4i| + |3 − 4i| + 2 + 5 + 2− 5 .

2. Determinat, i numerele reale a pentru care graficul funct, iei f : R → R,


f (x) = ax2 − (a + 1)x + a este situat deasupra axei Ox.

3. Rezolvat, i ı̂n mult, imea numerelor ı̂ntregi inecuat, ia log 3 x ≥ −3.


4

4. Câte numere naturale de două cifre au suma cifrelor mai mică cu 11 decât
produsul acestora?

5. În reperul cartezian xOy se consideră punctele A(−1, −2) s, i B(4, 8). Deter-
minat, i coordonatele punctului P ∈ (AB) pentru care 2P A = 3P B.

2020π 2020π
6. Calculat, i sin + cos .
3 3

SUBIECTUL al II-lea (30 de puncte)


2x + y = 0

1. Se consideră sistemul de ecuat, ii x + 2y + az = b , unde a, b ∈ R.

4ax + 5y + 2z = 0

a) Pentru b = 0, determinat, i a ∈ R pentru care sistemul are cel put, in o


solut, ie nenulă.
b) Rezolvat, i sistemul pentru a = 1 s, i b = 3.
c) Pentru b = 3, determinat, i a ∈ Z pentru care sistemul are solut, ie unică
(x0 , y0 , z0 ) s, i x0 , y0 , z0 ∈ Z.

2. Pe mult, imea numerelor reale se defines, te legea de compozit, ie asociativă


p
3
x∗y = x3 y 3 − x3 − y 3 + 2.

a) Arătat, i că legea ,,∗” admite element neutru.


b) Arătat, i că există un unic element x ∈ R care nu este simetrizabil ı̂n
raport cu legea ,,∗”.
c) Demonstrat, i că funct, ia f : R \ {1} → R∗ , f (x) = x3 − 1 este un
izomorfism de la grupul (R \ {1}, ∗) la grupul (R∗ , ·).
62 Costel BĂLCĂU

SUBIECTUL al III-lea (30 de puncte)

1. Se consideră funct, ia f : (−∞, 2) → R, f (x) = 2 ln(2 − x) − (1 − x)(3 − x).

a) Determinat, i ecuat, ia asimptotei verticale la graficului funct, iei f .


b) Arătat, i că funct, ia f este concavă.
c) Fie funct, iile g : R → R, g(x) = x2 − 4x + 3 s, i h : (−∞, 2) → R,
h(x) = ln(x − 2)2 . Demonstrat, i că graficele funct, iilor g s, i h au un
singur punct comun.
1−x
2. Se consideră funct, ia f : R → R, f (x) = .
ex
a) Dacă F este o primitivă a funct, iei f , arătat, i că F (x) ≤ F (1), pentru
orice x ∈ R.
Z 4 √
f ( x)
b) Calculat, i √ dx.
1 x
√n

n √
(n − 1) en−1 + (n − 2) en−2 + . . . + n e
c) Calculat, i lim .
n→∞ n2
Cuprins 63

Cuprins

PROBLEME PENTRU CONCURSURI 3

Probleme propuse 3

Rezolvarea problemelor din numărul anterior 11

ARTICOLE S, I NOTE MATEMATICE 31

Daniel JINGA
În legătură cu Problema 27710 din G.M.-B nr. 6-7-8/2019 . . . . . . . . . . 31

Dorin MĂRGHIDANU
O demonstrat, ie inductivă a Inegalităt, ii mediilor . . . . . . . . . . . . . . . 33

Florin STĂNESCU
Inegalităt, i pentru determinant, ii matricelor de ordinul 2 . . . . . . . . . . . 37

Leonard Mihai GIUGIUC s, i Do Huu Duc THINH


O rafinare a Inegalităt, ii lui Turkevich . . . . . . . . . . . . . . . . . . . . . 41

TESTE PENTRU EXAMENE 45

Costel ANGHEL s, i Florea BADEA


Teste pentru examenul de Evaluare Nat, ională . . . . . . . . . . . . . . . . . 45

Mihai Florea DUMITRESCU


Teste pentru examenul de Bacalaureat, specializarea s, tiint, e ale naturii . . . 53

Costel BĂLCĂU
Teste pentru examenul de Bacalaureat, specializarea matematică-informatică 58
Revistă sponsorizată de SC VISTORIA LUX SRL

S-ar putea să vă placă și